Board logo

标题: 费费feifei逻辑讲解版 [打印本页]

作者: capsicum    时间: 2011-7-5 17:38     标题: 费费feifei逻辑讲解版

[attach]12350[/attach]
作者: capsicum    时间: 2011-7-5 17:38

《费费逻辑宝典》详解版


Test 1

1. Although 90 percent of the population believes itself to be well informed about health care, only 20 percent knows enough about DNA. So apparently at least 80 percent of the population does not know enough about medical concepts to make well-informed personal medical choices or to make good public policy decisions about health care.

The argument’s reasoning is questionable because the argument fails to demonstrate that

A. those people who can understand news stories about DNA are able to make well-informed personal medical choices
B. more than 20 percent of the population needs to be well informed about health care for good public policy decisions about health care to be made
C. one’s being able to make well-informed personal medical choices ensures that one makes good public policy decisions about health care
D. an understanding of DNA is essential to making well-informed personal medical choices or to making good public policy decisions about health care
E. since 90 percent of the population believes itself to be well informed about health care, at least 70 percent of the population is mistaken in that belief.

答案:D
思路:本题的推论错误在于以偏概全,用对DNA的了解程度来考核well informed about health care的程度。据个例子,我们可否用掌握核物理知识的程度来考核对物理基本知识的掌握程度呢?显然不行!

A: 支持作者的观点
B: 无关
C: 无关
D: 本题的推论错误在于以偏概全,用对DNA的了解程度来考核well informed about health care的程度。据个例子,我们可否用掌握核物理知识的程度来考核对物理基本知识的掌握程度呢?显然不行!
E: 无关
作者: capsicum    时间: 2011-7-5 17:38

2. During the 1980’s, Japanese collectors were very active in the market for European art, especially as purchasers of nineteenth-century Impressionist paintings. This striking pattern surely reflects a specific preference on the part of many Japanese collectors for certain aesthetic attributes they found in nineteenth-century Impressionist paintings.

Which one of the following, if true, most strongly supports the explanation above?

A. Impressionist paintings first became popular among art collectors in Europe at the beginning of the twentieth century.
B. During the 1980’s, the Japanese economy underwent a sustained expansion that was unprecedented in the country’s recent history.
C. Several nineteenth-century Impressionist painters adopted certain techniques and visual effects found in Japanese prints that are highly esteemed in Japan.
D. During the 1960s and the 1970s, the prices of nineteenth-century Impressionist paintings often exceeded the prices of paintings by older European masters.
E. During the 1980’s, collectors from Japan and around the world purchased many paintings and prints by well-known twentieth-century Japanese artists.

答案:C
思路:
A. 无关
B. 无关
C. 作者先提出一个事实,然后指这个事实说明了日本收藏家对19世纪印象派画家的一些审美特性有着特别的偏爱。C直接举证说明原因:这是因为19世纪印象派画家使用了一些在日本备受推崇的技术和视觉效果。(Several nineteenth-century Impressionist painters adopted certain techniques and visual effects found in Japanese prints that are highly esteemed in Japan.)
D. 无关项, many Japanese collectors for certain aesthetic attributes 和D中的price没有任何关系
E. 无关
作者: capsicum    时间: 2011-7-5 17:39

3. Because of the recent recession in Country A most magazines published there have experienced decreases in advertising revenue, so much so that the survival of the most widely read magazines is in grave doubt. At the same time, however, more people in Country A are reading more magazines than ever before, and the number of financially successful magazines in Country A is greater than ever.

Which one of the following, if true, most helps to resole the apparent discrepancy in the information above?

A. Most magazines reduce the amount they charge for advertisement’s during a recession.
B. The audience for a successful television show far exceeds the readership of even the most widely read magazine.
C. Advertising is the main source of revenue only for the most widely read magazines; other magazines rely on circulation for their revenue.
D. Because of the recession, people in Country A have cut back on magazine subscriptions and are reading borrowed magazines.
E. More of the new general interest magazines that were launched this year in Country A have survived than survived in previous years.

参考答案:C

思路:explain题型的阅读要点是分歧主要在哪里。
A:无法解释后一现象
B:无关选项
C:抓住了分歧点:the most widely read magazines和financially successful magazines的区别
D:无法解释后一现象
E:无关选项(无关对比)
作者: capsicum    时间: 2011-7-5 17:39

4. The gray squirrel, introduced into local woodlands ten years ago, threatens the indigenous population of an endangered owl species, because the squirrels’ habitual stripping of tree bark destroys the trees in which the owls nest. Some local officials have advocated setting out poison for the gray squirrels. The officials argue that this measure, while eliminating the squirrels, would pose no threat to the owl population, since the poison would be placed in containers accessible only to squirrels and other rodents.

Which one of the following, if true, most calls into question the officials’ argument?

A. One of the species whose members are likely to eat the poison is the red squirrel, a species on which owls do not prey.
B. The owls whose nesting sites are currently being destroyed by the gray squirrels feed primarily on rodents.
C. No indigenous population of any other bind species apart from the endangered owls is threatened by the gray squirrels.
D. The owls that tare threatened build their nests in the tops of trees, but the gray squirrels strip away back from the trunks.
E. The officials’ plan entails adding the poison to food sources that are usually eaten by rodents but not by other animals.

参考答案:B
思路:本题的错误在于,作者提出一种解决问题的方法,但忽略了他的副作用或反作用,从而没有从根本上解决问题或引出其他更重要的问题。
A:无关 red squirrel
B:如果rodents是owls的主要食物,那毒死rodents将更严重的减少owls的数量。
C:无关 Here the concerned object is the owl, not the other birds
D:无关
E:无关选项 Without further information about the rodents, this choice does not bother the argument
作者: capsicum    时间: 2011-7-5 17:39

Questions 5-6
Sales manager: Last year the total number of meals sold in our company restaurants was much higher than it was the year before. Obviously consumers find our meals desirable.
Accountant: If you look at individual restaurants, however, you find that the number of meals sold actually decreased substantially at every one of our restaurants that was in operation both last year and the year before. The desirability of our meals to consumers has clearly decreased, given that this group of restaurants---the only ones for which we have sales figures that permit a comparison between last year and the year before---demonstrates a trend toward fewer sales.

5. If the sales figures cited by the accountant and the sales manager are both accurate, which one of the following must be true?

A. The company opened at least one new restaurant in the last two years.
B. The company’s meals are less competitive than they once were.
C. The quality of the company’s meals has not improved over the last two years.
D. The prices of the company’s meals have changed over the past two years.
E. The market share captured by the company’s restaurants fell last year.

参考答案:A
思路:简单数学计算题。个体收入(利润,成本等)x 数量=总数
A:个体收入减少,总收入上升,必然是数量增加。所赐选项正确。
遇到这种题,只要列出公式即可解决。
作者: capsicum    时间: 2011-7-5 17:40

6. Which one of the following, if true, most seriously calls into question the accountant’s argument?

A. The company’s restaurants last year dropped from their menus most of the new dishes that had been introduced the year before.
B. Prior to last year there was an overall downward trend in the company’s sales.
C. Those of the company’s restaurants that did increase their sales last year did not offer large discounts on prices to attract customers.
D. Sales of the company’s most expensive meal contributed little to the overall two-year sales increase.
E. Most of the company’s restaurants that were in operation throughout both last year and the year before are located in areas where residents experienced a severe overall decline in income last year.

Sales Manager: the total number of meals sold last year much higher à consumers desire our meals
Accountant: the number of meals sold by every restaurants operating these two years actually decreased last year from the year before (事实)à the desirability of our meals has decreased ( conclusion) à a trend toward fewer sales (conclusion)
1, 不同的取样,Sales Manager depends on the total number of meals sold in the company in the last year, while Accountant depends on each of 2-year-history restaurant;
2, 论据与结论之间没有必然的联系,却被当成充分条件。他因可以解释结论。

A. Reason to explain why the sale of each sample restaurant decreased last year. (无关)
B. 总公司前年的下降不能否定去年的下降,也不能支持。(无关)
C. 那些去年销量不降反增的公司是因为没有降价太多。不降反增的公司与下降的公司是两个不同的集合,描述对象不正确。如果我们推出那些下降的公司是因为降价太多,也只能是给出一个原因来解释为什么下降。 (无关)
D. 公司最贵的招牌菜并没有给公司带来销售的增加。也属于一个解释,解释为什么总量会下降。但与Accountant 的观点并没有直接的联系。既没有针对描述对象:2-year-history restaurant,也没有削弱论据与结论之间的必然联系。
E. 答案。针对描述对象:2-year-history restaurant,他因:a severe overall decline in income
时间的一致:last year.
作者: capsicum    时间: 2011-7-5 17:40

7. Problems caused by the leaching of pollutants from dumps and landfills are worst in countries with an annual per capita economic output of $4,000 to $5,000, and less severe for considerably poorer and considerably richer countries. This is so because pollution problems increase during the early stages of a country’s industrial development but then diminish as increasing industrial development generates adequate resources to tackle such problems. Therefore, problems caused by such leaching in Country X, where the annual per capita economic output is now $5,000, should begin to diminish in the next few years.

Which one of the following is an assumption on which the argument depends?

A. Within the next few years, Country X will impose a system of fines for illegal waste disposal by its industrial companies.
B. Countries surrounding Country X will reduce the amount of pollution that their factories release into the air and water.
C. Industrial development in Country X will increase in the next few years.
D. Country X will begin the process of industrialization in the next few years.
E. No other country with a similar amount of industrial development has pollution problems that are as severe as those in Country X.

Principle: Problems caused by the leaching of pollutants from dumps and landfills are worst in countries with an annual per capita economic output of $4,000 to $5,000, and less severe for considerably poorer and considerably richer countries.
Premise: (大前提)pollution problems increase during the early stages of a country’s industrial development but then diminish as increasing industrial development generates adequate resources to tackle such problems,用来支持最后的 conclusion: : problems in Country X should begin to diminish in the next few years.
Evidence: in Country X, where the annual per capita economic output is now $5,000,
Conclusion: problems in Country X should begin to diminish in the next few years.
是对 premise 的一次改写。
Hidden Assumption: The industrial development in Country X will increase in the next years so that its economic output will be more than $5,000, it passes over the early stage of industrial development and has adequate resources to tackle leaching problems.

题目问Assumption, 即必要条件。 只有这个必要条件成立,结论才能成立(A->B ) 。没有这个必要条件,结论是不可能成立的(非B-> 非A)

A. 高额的罚款并不是country X抯 problem will diminish. 的必要条件。因为如果由公司从自身的经济利益出发,宁愿选择接受罚款,则结论不成立。
B. out of scope
C. Correct Answer. 正符合我们的 Assumption.
D. Weaken the conclusion --- because that pollution problems increase during the early stages of a country’s industrial development (early stages == begin the process of industrialization)
E. 时间不一致,(一般现在式与begin to diminish in the next few years.)
比较对象不对。(Other country 与 Country X vs. Country X 现在 与 未来)
作者: capsicum    时间: 2011-7-5 17:42

8. The stable functioning of a society depends upon the relatively long-term stability of the goals of its citizens. This is clear from the fact that unless the majority of individuals have a predictable and enduring set of aspirations, it will be impossible for a legislature to craft laws that will augment the satisfaction of the citizenry, and it should be obvious that a society is stable only if its laws tend to increase the happiness of its citizens.

The claim that a society is stable only if its laws tend to increase the happiness of its citizens plays which one of the following roles in the argument?

A. It is the conclusion of the argument.
B. It helps to support the conclusion of the argument.
C. It is a claim that must be refuted if the conclusion is to be established.
D. It is a consequence of the argument.
E. It is used to illustrate the general principle that the argument presupposes.

这是一个典型的必要题型,题目之所以觉得有难度,
因为conclusion 在首而论证过程在后,而且还有概念的等价交换:to increase the happiness of its citizens ==to augment the satisfaction of the citizenry

Conclusion: The stable functioning of a society depends upon the relatively long-term stability of the goals of its citizens.

it should be obvious that a society is stable only if its laws tend to increase the happiness of its citizens.
A society is stable (a) à its laws tend to increase the happiness of its citizens. (b)

This is clear from the fact that unless the majority of individuals have a predictable and enduring set of aspirations, it will be impossible for a legislature to craft laws that will augment the satisfaction of the citizenry,
To craft laws that will augment the satisfaction of the citizenry (b)à the majority of individuals have a predictable and enduring set of aspirations, (c)

Therefore, A requires C. (the conclusion)
看懂文章后,不难得到正确答案B.
作者: capsicum    时间: 2011-7-5 17:42

9. Astronauts who experience weightlessness frequently get motion sickness. The astronauts see their own motion relative to passing objects, but while the astronauts are weightless their inner ears indicate that their bodies are not moving. The astronauts’ experience is best explained by the hypothesis that conflicting information received by the brain about the body’s motion causes motion sickness.

Which one of the following, if true, provides the strongest additional support for the hypothesis above?

A. During rough voyages ship passengers in cabins providing a view of the water are less likely to get motion sickness than are passengers in cabins providing no view.
B. Many people who are experienced airplane passengers occasionally get motion sickness.
C. Some automobile passengers whose inner ears indicate that they are moving and who have a clear view of the objects they are passing get motion sickness.
D. People who have aisle seats in trains or airplanes are as likely to get motion sickness as are people who have window seats.
E. Some astronauts do not get motion sickness even after being in orbit for several days.

答案:A
思路:搭桥法,建立A与B的联系。仔细读题可知:这种运动性疾病是由于大脑接受了不同的矛盾的信息。有两个信息源:一个来自内耳(静止),一个来自眼睛(运动)。
选项中只有A,C,D涉及这样两个信息源。
A指出接受两种不同的信息较容易获得运动性疾病比起获得两种一致的信息。C:虽有两种信息,但两者是一致的(都是运动的)。
C, D:指出接受两种一致的信息较容易获得运动性疾病比起获得两种不同的信息。与题目的例子相反。所以只有A是正确选项。
作者: capsicum    时间: 2011-7-5 17:42

10. In order to pressure the government of Country S to become less repressive, some legislators in Country R want to ban all exports from R to S. Companies in R that manufacture telecommunication equipment such as telephones and fax machines have argued that exports of their products should be exempted from the ban, on the ground that it is impossible for a country to remain repressive when telecommunication equipment is widely available to the population of that country.

Which one of the following is an assumption on which the argument given by the manufacturers depends?

A. The government of S has recently increased the amount of telecommunication equipment it allows to be imported into the country.
B. The telecommunication equipment that would be imported into S if the exemption were to be granted would not be available solely to top development officials in S.
C. A majority of the members of R’s legislature do not favor exempting telecommunication equipment from the ban on exports to Country S.
D. Of all exports that could be sent to Country S, telecommunication equipment would be the most effective in helping citizens of S oppose that country’s repressive government.
E. Without pressure from Country R, the government of S would be able to continue repressing its citizens indefinitely.

The correct answer is B.

See: Assumption question

Think: I have to find the conclusion, then the evidence, and from them figure out the central assumption.

Do:
Conclusion: The “what” of an argument

In this case:
In order to pressure the government of Country S to become less repressive, some legislators in Country R want to ban all exports from R to S.

Evidence: The “Why” of an argument

In this case:
It’s impossible for a country to remain repressive when telecommunication equipment is widely available to the population of that country.

Assumption: Facts or ideas, not stated in the passage. The assumption must be true if the argument is to be considered valid. The assumption typical directly links the evidence with the conclusion. The best way to tackle it is to negate it.

In this case:
The telecommunication equipment that would be imported into S if the exemption were to be granted would not be available solely to top development officials in S.

If we negate the assumption, the conclusion can’t come to pass.
:
The telecommunication equipment that would be allowed to export into S (exempted) would be AVAILABLE SOLELY to top development officials in S, then the proposal to pressure the government of Country S to become less repressive by using ban will be failed. As a result, the argument that manufacture gave that telecommunication equipment such as telephones and fax machines have argued that exports of their products should be exempted from the ban will be failed.
作者: capsicum    时间: 2011-7-5 17:43

11. The average cable television company offers its customers 50 channels, but new fiber-optic lines will enable telephone companies to provide 100 to 150 television channels to their customers for the same price as cable companies charge for 50. Therefore, the new companies will displace cable companies within a few years.

Which of the following, if true, most helps to strengthen the argument?

A. The initial cost per household of installing new fiber-optic television service will exceed the current cost of installing cable television service.
B. The most popular movies and programs on channels carried by cable companies will also be offered on channels carried by the fiber-optic lines owned by the telephone companies.
C. Cable television companies will respond to competition from the telephone companies by increasing the number of channels they offer.
D. Some telephone companies own cable companies in areas other than those in which they provide telephone services.
E. The new fiber-optic services offered by telephone companies will be subject to more stringent governmental programming regulations than those to which cable companies are now subject.

The correct answer is B.

See: Strengthen question

Think: I need the choice that confirms the central assumption. In other words, Strengthen the argument = Strengthen the assumption.

Do:
Conclusion: The “what” of an argument

In this case:
Cable companies will be displaced by the new companies within a few years.
.
Evidence: The “Why” of an argument

In this case:
The average cable television company offers its customers 50 channels, but new fiber-optic lines will enable telephone companies to provide 100 to 150 television channels to their customers for the same price as cable companies charge for 50.

Assumption: Facts or ideas, not stated in the passage. The assumption must be true if the argument is to be considered valid. The assumption typical directly links the evidence with the conclusion. The best way to tackle it is to negate it.

In this case:
The channels, if not all, the most of them must be overlapped from both providers.

The choice B, only B, confirms the central assumption. So it’s the correct answer.
作者: capsicum    时间: 2011-7-5 17:43

12. The only physical factor preventing a human journey to Mars has been weight. Carrying enough fuel to propel a conventional spacecraft to Mars and back would make even the lightest craft too heavy to be launched from Earth. A device has recently been invented, however, that allows an otherwise conventional spacecraft to refill the craft’s fuel tanks with fuel manufactured from the Martian atmosphere for the return trip. Therefore, it is possible for people to go to Mars in a spacecraft that carries this device and then return.

Which one of the following is an assumption on which the argument depends?

A. The amount of fuel needed for a spacecraft to return from Mars is the same as the amount of fuel needed to travel from Earth to Mars.
B. The fuel manufactured from the Martian atmosphere would not differ in composition from the fuel used to travel to Mars.
C. The device for manufacturing fuel from the Martian atmosphere would not take up any of the spaceship crew’s living space.
D. A conventional spacecraft equipped with the device would not be appreciably more expensive to construct than current spacecraft typically are.
E. The device for manufacturing fuel for the return to Earth weighs less than the tanks of fuel that a conventional spacecraft would otherwise need to carry from Earth for the return trip.

The correct answer is (E)

See: Assumption question

Think: I have to find the conclusion, then the evidence, and from them figure out the central assumption.

Do:
Conclusion: The “what” of an argument

In this case:
It is possible for people to go to Mars in a spacecraft that carries this device and then return.

Evidence: The “Why” of an argument

In this case:
E1:
The only physical factor preventing a human journey to Mars has been weight. Carrying enough fuel to propel a conventional spacecraft to Mars and back would make even the lightest craft too heavy to be launched from Earth.
E2:
A device has recently been invented, however, that allows an otherwise conventional spacecraft to refill the craft’s fuel tanks with fuel manufactured from the Martian atmosphere for the return trip

Assumption: Facts or ideas, not stated in the passage. The assumption must be true if the argument is to be considered valid. The assumption typical directly links the evidence with the conclusion. The best way to tackle it is to negate it.

In this case:
The device for manufacturing fuel for the return to Earth weighs less than the tanks of fuel that a conventional spacecraft would otherwise need to carry from Earth for the return trip.


If we negate the assumption, the conclusion can’t come to pass.

The device for manufacturing fuel for the return to Earth weighs more than the tanks of fuel that a conventional spacecraft would otherwise need to carry from Earth for the return trip, then the conclusion can’t come to pass in new invented device term.
作者: capsicum    时间: 2011-7-5 17:43

13. In 1712 the government of Country Y appointed a censor to prohibit the publication of any book critical of Country Y’s government; all new books legally published in the country after 1712 were approved by a censor. Under the first censor, one half of the book manuscripts submitted to the censor were not approved for publication. Under the next censor, only one quarter of the book manuscripts submitted were not approved, but the number of book manuscripts that were approved was the same under both censors. If the statements in the passage are true, which one of the following can be properly concluded from them?

A. More books critical of Country Y’s governments were published before the appointment of the first censor than after it.
B. The first censor and the second censor prohibited the publication of the same number of book manuscripts.
C. More book manuscripts were submitted for approval to the first censor than to the second.
D. The second censor allowed some book manuscripts to the published that the first censor would have considered critical of Country Y’s government.
E. The number of writers who wrote unpublished manuscripts was greater under the first censor than under the second.

参考思路:

同5,又是一道计算题。关键还是列出公式。
the number of book manuscripts that were approved was the same under both censors.
Numbers of the first censor checked x (1-50%) = Numbers of the second censor x (1-25%)
So
(C) More book manuscripts were submitted for approval to the first censor than to the second.
作者: capsicum    时间: 2011-7-5 17:44

14. If the government increases its funding for civilian scientific research, private patrons and industries will believe that such research has become primarily the government’s responsibility. When they believe that research is no longer primarily their responsibility, private patrons and industries will decrease their contributions toward research. Therefore, in order to keep from depressing the overall level of funding for civilian scientific research, the government should not increase its own funding.

Which one of the following is an assumption on which the argument relies?

A. Governments should bear the majority of the financial burden of funding for civilian scientific research.
B. Any increase in government funding would displace more private funding for civilian scientific research than it would provide.
C. Private donations toward research are no longer welcomed by researchers whose work receives government funding.
D. Civilian scientific research cannot be conducted efficiently with more than one source of funding.
E. funding for civilian scientific research is currently at the highest possible level.

参考思路:
本题指出private funding for civilian scientific research和government funding是一增一减的关系。如果两者的增长数量是一样的,则总量不会随其中之一而变化。而如果一方的变化是另一方变化量的系数倍,则总量将会受到影响
(B) Any increase in government funding would displace more private funding for civilian scientific research than it would provide.
这个选项指出了第二钟变化趋势。Therefore, in order to keep from depressing the overall level of funding for civilian scientific research, the government should not increase its own funding. 我们应该假设Any increase in government funding would displace more private funding for civilian scientific research than it would provide.。 b is correct.
作者: capsicum    时间: 2011-7-5 17:44

15. Dental researcher: Filling a cavity in a tooth is not a harmless procedure: it inevitably damages some of the healthy parts of the tooth. Cavities are harmful only if the decay reaches the nerves inside the tooth, and many cavities, if left untreated, never progress to that point. Therefore, dentists should not fill a cavity unless the nerves inside the tooth are in imminent danger from that cavity.

Which one of the following principles, if valid, most strongly supports the researcher’s reasoning?

A. Dentists should perform any procedure that is likely to be beneficial in the long term, but only if the procedure does not cause immediate damage.
B. Dentists should help their patients to prevent cavities rather than waiting until cavities are present to begin treatment.
C. A condition that is only potentially harmful should not be treated using a method that is definitely harmful.
D. A condition that is typically progressive should not be treated using methods that provide only temporary relief.
E. A condition that is potentially harmful should not be left untreated unless it can be kept under constant surveillance.

答案:C
思路:搭桥法,建立A与B的联系作者指出:补牙对牙齿的健康部分是一定有害的。而牙洞只有在已经露出神经后才是有害的。可是这种情形几乎不可能出现。因此牙医不应该随便给病人补牙除非牙龈神经已经露出。
A, 提到无关信息:beneficial in the long term。
B, 与题目相反
C, 有效的建立了两者的关系,即对于潜在的伤害不应用必然带来其他伤害的方法来治疗。
D, 无关信息排除
E, 没有建立起有效的连接。
作者: capsicum    时间: 2011-7-5 17:44

16. The axis of Earth’s daily rotation is tilted with respect to the plane of its orbit at an angle of roughly 23 degrees. That angle can be kept fairly stable only by the gravitational influence of Earth’s large, nearby Moon. Without such a stable and moderate axis tilt, a planet’s climate is too extreme and unstable to support life. Mars, for example, has only very small moons, tilts at wildly fluctuating angles, and cannot support life.

If the statements above are true, which one of the following must also be true on the basis of them?

A. If Mars had a sufficiently large nearby moon, Mars would be able to support life.
B. If Earth’s Moon were to leave Earth’s orbit, Earth’s climate would be unable to support life.
C. Any planet with a stable, moderate axis tilt can support life.
D. Gravitational influences other than moons have little or no effect on the magnitude of the tilt angle of either Earth’s or Mars’s axis.
E. No planet that has more than one moon can support life.

参考思路:
A only by B。 B是A的必要条件, 非B推出非A。
If Earth’s Moon were to leave Earth’s orbit -----〉That angle cannot be kept fairly stable ---à
a planet’s climate is too extreme and unstable to support life.( based on Without such a stable and moderate axis tilt, a planet’s climate is too extreme and unstable to support life.)
(B) If Earth’s Moon were to leave Earth’s orbit, Earth’s climate would be unable to support life.
Is correct!
此题为典型的逻辑关系推理题。
作者: capsicum    时间: 2011-7-5 17:44

17. Psychologist: Some astrologers claim that our horoscopes completely determine our personalities, but this claim is false, I concede that identical twins---who are, of course, born at practically the same time---often do have similar personalities. However, birth records were examined to find two individuals who were born 40 years ago on the same day and at exactly the same time---one in a hospital in Toronto and one in a hospital in New York. Personalities of these two individuals are in fact different.

Which one of the following is an assumption on which the psychologist’s argument depends?

A. Astrologers have not subjected their claims to rigorous experimentation.
B. The personality differences between the two individuals cannot be explained by the cultural difference between Toronto and New York.
C. The geographical difference between Toronto and New York did not result in the two individuals having different horoscopes.
D. Complete birth records for the past 40 years were kept at both hospitals.
E. Identical twins have identical genetic structures and usually have similar home environments.

答案:C
思路: Psychologist举例否原观点,举的反例必定是在原观点相同的前提情况下产生了与原观点不同的结论。原观点的前提情况是相同星座,结论是性格相同。反例的前提情况需要与原观点的一致,所以即是不管多伦多还是纽约生的星座是一样的。
A, 是无关选项,占星家们做没做实验对原文半点影响没有。
B, 是无关选项,心理学家们驳的是星象,跟文化没关系。
D, 是废话,重复了原文的背景。
E, 也是废话,重复了原文的背景。
作者: capsicum    时间: 2011-7-5 17:45

18. Modern navigation systems, which are found in most of today’s commercial aircraft, are made with low-power circuitry, which is more susceptible to interference than the vacuum-tube circuitry found in older planes. During landing, navigation systems receive radio signals from the airport to guide the plane to the runway. Recently, one plane with low-power circuitry veered off course during landing, its dials dimming, when a passenger turned on a laptop computer. Clearly, modern aircraft navigation systems are being put at risk by the electronic devices that passengers carry on board, such as cassette players and laptop computers.

Which one of the following, if true, LEAST strengthens the argument above?

A. After the laptop computer was turned off, the plane regained course and its navigation instruments and dials returned to normal.
B. When in use all electronic devices emit electromagnetic radiation, which is known to interfere with circuitry.
C. No problems with navigational equipment or instrument dials have been reported on flights with no passenger-owned electronic devices on board.
D. Significant electromagnetic radiation from portable electronic devices can travel up to eight meters, and some passenger seats on modern aircraft are located within four meters of the navigation systems.
E. Planes were first equipped with low-power circuitry at about the same time portable electronic devices became popular.

答案:E
思路: E是无关选项,同时装备无法支持飞机航行受电子设备干扰。其他选项都能支持。
A, 加强选项,关了电玩后,飞机就正常了,说明电玩有影响。
B, 加强选项,直接说电玩影响飞机电路。
C, 没有电玩,飞机没影响,相对加强力度弱些,但比起E来也能勉强算做加强。
D, 加强选项,电玩八米有影响,有些乘客就坐在电路八米内。
作者: capsicum    时间: 2011-7-5 17:45

TEST 2

19. Jane: Television programs and movies that depict violence among teenagers are extremely popular. Given how influential these media are, we have good reason to believe that these depictions cause young people to engage in violent behavior. Hence, depictions of violence among teenagers should be prohibited from movies and television programs, if only in those programs and movies promoted to young audiences.

Maurice: But you are recommending nothing short of censorship! Besides which, your claim that television and movie depictions of violence cause violence is mistaken: violence among young people predates movies and television by centuries.

Which one of the following, if true, most strengthens Jane’s argument?

A. The most violent characters depicted in movies and on television programs are adult characters who are portrayed by adult actors.
B. The movies that have been shown to have the most influence on young people’s behavior are those that are promoted to young audiences.
C. The people who make the most profits in the movie and television industry are those who can successfully promote their work to both young and old audiences.
D. Many adolescents who engage in violent behavior had already displayed such behavior before they were exposed violence in movies.
E. Among the producers who make both movies and television programs, many voluntarily restrict the subject matter of films directed toward young audiences.
答案:B
思路:J说由于恶少们很多是被电影带坏的,所以带暴力的镜头应该除掉。要支持,就要建立连接,说这些电影对孩儿们影响太大了,所以选B。

A, 无关选项,成人角色原文没提过。
C, 无关选项,搞影视的大腕跟原文也没关系。
D, 削弱,说那帮小孩们从来就是暴力狂,跟影视没关系。
E, 比较模糊,说是制作人们比较自觉,一般都删掉暴力镜头,可以说无关,也可以说削弱,但是绝对不是加强。
作者: capsicum    时间: 2011-7-5 17:45

20. A rise in the percentage of all 18-year-olds who were recruited by the armed services of a small republic between 1980 and 1986 correlates with a rise in the percentage of young people who dropped out of high school in that republic. Since 18-year-olds in the republic are generally either high school graduates or high school dropouts, the correlation leads to the conclusion that the republic’s recruitment rates for 18-year-olds depend substantially on recruitment rates for high school dropouts.

Which one of the following statements, if true, most weakens the argument?

A. A large number of 18-year-old high school graduates were recruited for the republic’s armed services in 1986 than in 1980.
B. Many of the high-technology systems used by the republic’s armed services can be operated only by individuals who have completed a high school education.
C. Between 1980 and 1986 the percentage of high school graduates among 18-year-olds recruited in the republic rose sharply.
D. Personnel of the republic’s armed services are strongly encouraged to finish their high school education.
E. The proportion of recruits who had completed at least two years of college education was greater in 1986 than in 1980.

参考答案:C
思路:题目的核心是要否定percentage of dropouts 与 percentage of recruitment的因果关系
A:典型的混淆相对数概念和绝对数概念,文中比较的是比率,选项给的是绝对数;
B:无关选项,和题目的核心比率没有太大关系。Tips: 注意many... only by.....这种表述。
C:正确,指出结论的错误属于false dilemma。
D:无关选项。此类错误属于false appeal。tips:注意.....(机构或人)strongly encourage something
E:无关选项,percentage of those completed college education和题目的核心比率没有太大关系。
作者: capsicum    时间: 2011-7-5 17:46

21. Historian: We can learn about the medical history of individuals through chemical analysis of their hair. It is likely, for example, that Isaac Newton’s psychological problems were due to mercury poisoning; traces of mercury were found in his hair. Analysis is now being done on a lock of Beethoven’s hair. Although no convincing argument has shown that Beethoven ever had a venereal disease, some people hypothesize that venereal disease caused his deafness. Since mercury was commonly ingested in Beethoven’s time to treat venereal disease, if researchers find a trace of mercury in his hair, we can conclude that this hypothesis is correct.

Which one of the following is an assumption on which the historian’s argument depends?

A. None of the mercury introduced into the body can be eliminated.
B. Some people in Beethoven’s time did not ingest mercury.
C. Mercury is an effective treatment for venereal disease.
D. Mercury poisoning can cause deafness in people with venereal disease.
E. Beethoven suffered from psychological problems of the same severity as Newton’s.

There should be 2 assumptions to support the hypothesis. The 1st assumption of the author is that only venereal disease will be treated by mercury in Beethoven’s time. The 2nd assumption is only some people in Beethoven's time will ingest mercury, if all people ingest mercury, the evidence is not useful. So, some people in Beethoven's time did not ingest mercury. The answer should be B.

A, E have nothing to do with the argument.
C is not necessary, 'Since mercury was commonly ingested in Beethoven’s time to treat venereal disease' is enough.
D is wrong: according to the author, it is the venereal disease that will cause the deafness, not the mercury poison.
作者: capsicum    时间: 2011-7-5 17:46

22. In 1992, a major newspaper circulated throughout North American paid its reporters an average salary paid by its principle competitors to their reporters. An executive of the newspaper argued that this practice was justified, since any shortfall that might exist in the reporters’ salaries is fully compensated by the valuable training they receive through their assignments.

Which one of the following, if true about the newspaper in 1992, most seriously undermines the justification offered by the executive?

A. Senior reporters at the newspaper earned as much as reporters of similar stature who worked for the newspaper’s principle competitors.
B. Most of the newspaper’s reporters had worked there for more than ten years.
C. The circulation of the newspaper had recently reached a plateau, after it had increased steadily throughout the 1980s.
D. The union that represented reporters at the newspaper was different from the union that represented reporters at the newspaper’s competitors.
E. The newspaper was widely read throughout continental Europe and Great Britain as well as North America.

参考答案:B
思路:否定low salaries is fully compensated by training
A:与结论无关
B:10年工作经验表示training的作用不大。
C:与结论无关。
D:与结论无关。
E:与结论无关。
评论:此题较难,只能用排除法作。正确选项也比较含糊,需要推理,这种题在gmat中出现的不多。
作者: capsicum    时间: 2011-7-5 17:46

23. On a certain day, nine scheduled flights on Swift Airlines were canceled. Ordinarily, a cancellation is due to mechanical problems with the airplane scheduled for a certain flight. However, since it is unlikely that Swift would have the mechanical problems with more than one or two scheduled flights on a single day, some of the nine cancellations were probably due to something else.
The argument depends on which one of the following assumptions?

A. More than one or two airplanes were scheduled for the nine canceled flights.
B. Swift Airlines has fewer mechanical problems than do other airlines of the same size.
C. Each of the canceled flights would have been longer than the average flight on Swift Airlines.
D. Swift Airlines had never before canceled more than one or two scheduled flights on a single day.
E. All of the airplanes scheduled for the canceled flights are based at the same airport.

参考答案:A 采用取非法。
思路:否定More than one or two airplanes were scheduled for the nine canceled flights.将削弱作者的结论。

A:取非削弱
B:无关。
C:无关。
D:无关。
E:无关。
作者: capsicum    时间: 2011-7-5 17:47

24. The interstitial nucleus, a sub-region of the brain’s hypothalamus, is typically smaller for male cats than for female cats. A neurobiologist performed autopsies on male cats who died from disease X, a disease affecting no more than .05 percent of male cats, and found that these male cats had interstitial nuclei that were as large as those generally found in female cats. Thus, the size of the interstitial nucleus determines whether or not male cats can contract disease X.

Which one of the following statements, if true, most seriously weakens the argument?

A. No female cats have been known to contract disease X, which is a subtype of disease Y.
B. Many male cats that contract disease X also contract disease Z, the cause of which is unknown.
C. the interstitial nuclei of female cats who contact disease X are larger than those of female cats who do not contract disease X.
D. Of 1,000 autopsies on male cats that did not contract disease X, 5 revealed interstitial nuclei larger than those of the average male cat.
E. The hypothalamus is known not to be causally linked to disease Y, and disease X is a subtype of disease Y.

E is the best answer. Disease X is the subtype is disease Y, if disease Y has no link with hypothalamus, X must have no link with hypothalamus.

A, C talked about the female cat, that can't weaken the argument.
B talked about another disease Z, doesn't weaken the argument.
D just shows some data, which cannot weaken the argument.
作者: capsicum    时间: 2011-7-5 17:47

25. There should be a greater use of gasohol. Gasohol is a mixture of alcohol and gasoline, and has a higher octane rating and fewer carbon monoxide emissions than straight gasoline. Burning gasohol adds no more carbon dioxide to the atmosphere than plants remove by photosynthesis.
Each of the following, if true, strengthens the argument above EXCEPT:

A. Cars run less well on gasoline than they do on gasohol.
B. Since less gasoline is needed with the use of gasohol, an energy shortage is less likely.
C. Cars burn on the average slightly more gasohol per kilometer than they do gasoline.
D. Gasohol is cheaper to produce and hence costs less at the pump than gasoline.
E. Burning gasoline adds more carbon dioxide to the atmosphere than plants can remove.

答案是C。解释:首先领会本argument的结论是:There should be a greater use of gasohol。根据提议,要选支持选项,即选出支持多使用gasohol的选项:
A: 用gasoline的汽车不如用gasohol的汽车好,起到支持作用
B: 说明使用gasohol能够降低发生能源短缺的可能性,当然起到支持作用
C: 汽车行驶一公里平均需要使用的gasohol比gasoline多,这说明了gasohol的一项劣势,不能起到支持作用
D: gasohol比gasoline具有较低的成本,支持。
E: 燃烧gasoline产生的二氧化碳比植物能消化的多,而argument中说gasohol产生的二氧化碳不比植物光合作用时能移除的多,所以优势显而易见。
作者: capsicum    时间: 2011-7-5 17:47

26. Barnes: The two newest employees at this company have salaries that are too high for the simple tasks normally assigned to new employees and duties that are too complex for inexperienced workers. Hence, the salaries and the complexity of the duties of these two newest employees should be reduced.

Which one of the following is an assumption on which Barnes’s argument depends?

A. The duties of the two newest employees are not less complex than any others in the company.
B. It is because of the complex duties assigned that the two newest employees are being paid more than is usually paid to newly hired employees.
C. The two newest employees are not experienced at their occupations.
D. Barnes was not hired at a higher-than-average starting salary.
E. The salaries of the two newest are no higher than the salaries that other companies pay for workers with a similar level of experience.

答案是C。解释:
A: 不是假设
B: 这似乎在为两个新雇员拿高工资找理由,所以不能支持结论。
C: 正确。因为原文中说对于不熟练的员工来说,工资太高了; 并且要降低工作的复杂程度。如果员工胜任的话,why bother? 所以,重要假设就是这些员工在岗位上不胜任。
D: 这个选项太离题了,而且似乎有贬低Barnes人格之嫌。
E: 这个起到了削弱结论的作用,所以不是assumption。
作者: capsicum    时间: 2011-7-5 17:47

27. These days, drug companies and health professionals alike are focusing their attention on cholesterol in the blood. The more cholesterol we have in our blood, the higher the risk that we shall die of a heart attack. The issue is pertinent since heart disease kills more North Americans every year than any other single cause. At least three factors---smoking, drinking, and exercise—can each influence levels of the cholesterol in the blood.

Which one of the following can be properly concluded from the passage?

A. If a person has low blood cholesterol, then that person’s risk of fatal heart disease is low.
B. Smoking in moderation can entail as great a risk of fatal heart disease as does heavy smoking.
C. A high-cholesterol diet is the principal cause of death in North America.
D. The only way that smoking increases one’s risk of fatal heart disease is by influencing the levels of cholesterol in the blood.
E. The risk of fatal heart disease can be altered by certain changes in lifestyle.

答案是E。解释:
A. 原文中说,胆固醇越高,生心脏病的可能性越大。A选项的内容是否命题,不一定成立。
B. 原文中没有提到此相关信息。
C. 显然不对,因为文中结尾明显说抽烟、饮酒和缺少运动也可以影响血液中胆固醇水平,所以不能保证C是正确的。
D. 文中没有说是only way哦。
E. 就是它了。可以通过调整抽烟、饮酒、运动等生活方式来减少心脏病的可能性。
作者: capsicum    时间: 2011-7-5 17:48

28. Ordinary mountain sickness, a common condition among mountain climbers, and one from which most people can recover, is caused by the characteristic shortage of oxygen in the atmosphere at high altitudes. Cerebral edema, a rarer disruption of blood circulation in the brain that quickly becomes life-threatening if not correctly treated from its onset, can also be caused by a shortage of oxygen. Since the symptoms of cerebral edema resemble those of ordinary mountain sickness, cerebral edema is especially dangerous at high altitudes.

Which one of the following is an assumption on which the argument depends?

(A) The treatment for ordinary mountain sickness differs from the treatment for cerebral edema.
(B) Cerebral edema can cause those who suffer from it to slip into a coma within a few hours.
(C) Unlike cerebral edema, ordinary mountain sickness involves no disruption of blood circulation in the brain.
(D) Shortage of oxygen at extremely high altitude is likely to affect thinking processes and cause errors of judgment.
(E) Most people who suffer from ordinary mountain sickness recover without any special treatment.

答案选A
这道题目先讲了ordinary mountain sickness怎么样的,然后又讲了cerebral edema是怎么样,它们两者有共同点就是symptoms of cerebral edema resemble those of ordinary mountain sicknes,所以如果不当心的话,就容易把cerebral edema 看成是ordinary mountain sickness那样的话is especially dangerous at high altitudes.
A. 选项,提出了一个假设,就是他们两者的医疗方法不同,因为如果相同的话,用治疗普通登山症的方法也可以治cerebral edema,就不会有危险了
B. 无关选项
C. 讲普通的登山症怎样,文中已经讲了,不是什么前提
D. 无关选项
E. 无关选项

The answer should be A,
the mountain climber know very well how to treat ordinary mountain sickness, since the symptoms are the alike, if they treat cerebral edema the same way as they treat ordinary mountain sickness, the danger of cerebral edema would be especially dangerous at high altitudes.

B,C :the description of B,C doesn't show any difference on high altitudes.
D : 'thinking processes and cause errors of judgment' were never mentioned in the description. It could be an answer, but the not so good as A.
E didn't answer why 'cerebral edema is especially dangerous at high altitudes'.
作者: capsicum    时间: 2011-7-5 17:48

29. The price of a full-fare coach ticket from Toronto to Dallas on Breezeway Airlines is the same today as it was a year ago, if inflation is taken into account by calculating prices in constant dollars. However, today 90 percent of the Toronto-to-Dallas coach tickets that Breezeway sells are discount tickets and only 10 percent are full-fare tickets, whereas a year ago half were discount tickets and half were full-fare tickets. Therefore, on average, people pay less today in constant dollars for a Breezeway Toronto-to-Dallas coach ticket than they did a year ago.

Which one of the following, if assumed, would allow the conclusion above to be properly drawn?

(A) A Toronto-to-Dallas full-fare coach ticket on Breezeway Airlines provides ticket-holders with a lower level of service today than such a ticket provided a year ago.
(B) A Toronto-to-Dallas discount coach ticket on Breezeway Airlines costs about the same in constant dollars as they did a year ago.
(C) All full-fare coach tickets on Breezeway Airlines cost the same in constant dollars as they did a year ago.
(D) The average number of coach passengers per flight that Breezeway Airlines carries from Toronto to Dallas today is higher than the average number per flight a year ago.
(E) The criteria that Breezeway Airlines uses for permitting passengers to buy discount coach tickets on the Toronto-to-Dallas route are different today than they were a year ago.

答案选B
本题所讲的,无非就是,people pay less today in constant dollars,而得出这个结论的证据是,以前10%有打折,现在一半有打折,那么,这个前提就是,他们的价格是一样的,不然,如果涨价,就不可能付的少了
A,service,无关
C,full-fare coach tickets cost the same 不等于打折后的也一样
D,无关
E,无关
作者: capsicum    时间: 2011-7-5 17:48

30. Combustion of gasoline in automobile engines produces benzene, a known carcinogen. Environmentalists propose replacing gasoline with methanol, which does not produce significant quantities of benzene when burned. However, combustion of methanol produces formaldehyde, also a know carcinogen. Therefore the environmentalists’ proposal has little merit.

Which one of the following, if true, most supports the environmentalist’ proposal?

A. The engines of some automobiles now on the road burn diesel fuel rather than gasoline.
B. Several large research efforts are under way to formulate cleaner-burning types of gasoline.
C. In some regions, the local economy is largely dependent on industries devoted to the production and distribution of automobile fuel.
D. formaldehyde is a less potent carcinogen than benzene.
E. Since methanol is water soluble, methanol spills are more damaging to the environment than gasoline spills.

选D
题目中Environmentalists propose replacing gasoline with methanol,但有些人认为combustion of methanol produces formaldehyde, also a know carcinogen.所以是不可取
怎样支持Environmentalists的观点呢,就是要说formaldehyde的好
D,说出formaldehyde 比benzene有 less potent carcinogen 那自然这个就是优先选择了
作者: capsicum    时间: 2011-7-5 17:49

Questions 31-32
Political opinion and analysis outside the mainstream rarely are found on television talk shows, and it might be thought that this state of affairs is a product of the political agenda of the television stations themselves. In fact, television stations are driven by the same economic forces as sellers of more tangible goods. Because they must attempt to capture the largest possible share of the television audience for their shows, they air only those shows that will appeal to large numbers of people. As a result, political opinions and analyses aired on television talk shows are typically bland and innocuous.

31. An assumption made in the explanation offered by the author of the passage is that

A. most television viewers cannot agree on which elements of a particular opinion or analysis are most disturbing.
B. there are television viewers who might refuse to watch television talk shows that they knew would be controversial and disturbing.
C. each television viewer holds some opinion that is outside the political mainstream, but those opinions are not the same for everyone.
D. there are television shows on which economic forces have an even greater impact than they do on television talk shows.
E. the television talk shows of different stations resemble one another in most respects.

B is the best answer.

先抽象原文:TV station的政治talk show是受经济利益驱使,看谁能capture the largest possible viewers,而不是TV station’s political agenda,所以,TV station的政治Talk show是bland and innocuous (柔和的和不得罪人的)。问explanation的assumption:
大部分viewer不能就opinion or analysis的哪部分最disturbing达成一致。能不能就most disturbing达成一致与收视率及bland and innocuous无关,并有一点weaken的味道。
tv reviewers拒绝看controversial and disturbing的talk show;这正好在“capture the largest possible viewers”与“bland and innocuous” 架了一座桥梁。So B is the best answer.
文章讨论的重点不是outside the political mainstream, 所以是无关项。
其他talk show 的economic forces无关。
different stations无关。
作者: capsicum    时间: 2011-7-5 17:49

32. The explanation offered by the author of the passage makes the assumption that

A. television station executives usually lack a political agenda of their own
B. bland and innocuous political opinions and analyses are generally in the mainstream
C. political analysts outside the mainstream are relatively indifferent to the effect their analyses have on television viewers
D. most television viewers are prepared to argue against allowing the expression of political opinions and analyses with which they disagree
E. the political opinions of television station executives are not often reflected in the television shows their stations produce

the best answer is B.

A. tv station executive is irrelevant to the explanation.
B. Yes. Because “political opinion and analysis outside the mainstream rarely found on tv talk show” , and at last the author concludes that“political opinions and analysis aired on tv talk shows are bland and innocuous”。
C. The passage doesn’t infer “political analysts outside the mainstream are relatively indifferent”, but assumes that “it might be thought…. of tv station themselves”. So C is irrelevant to the explanation.
D. This is a confusing choice. “Argue against” doesn’t indicate it will affect the tv viewers to see the talk show. It is the attitude, not the effect or result of this attitude. So it will not help the author to reason the conclusion.
E. Obviously, “the political opinion of tv station executive” is not mentioned in this passage and we can’t evaluate this situation before we get some relative information. So it is irrelevant.
作者: capsicum    时间: 2011-7-5 17:50

Questions 33-34
Conservationist: The population of a certain wildflower is so small that the species is headed for extinction. However, this wildflower can cross-pollinate with a closely related domesticated daisy, producing viable seeds. Such cross-pollination could result in a significant population of wildflower-daisy hybrids. The daisy should therefore be introduced into the wildflower’s range, since although the hybrid would differ markedly from the wildflower, hybridization is the only means of preventing total loss of the wildflower in its range.

33. Which one of the following principles, if valid, most helps to justify the conservationist’s reasoning?

(A) It is better to take measures to preserve a valued type of organism, even if those measures are drastic, than to accept a less valuable substitute for the organism.
(B) It is better to preserve a type of organism that is in danger of extinction, even if surviving organisms of that type are not vigorous, than to allow something more vigorous to replace it.
(C) It is better to change a type of organism that would otherwise be lost, even if the changes are radical, than to lose it entirely.
(D) It is better to destroy one of two competing types of organisms, even if both are irreplaceable, than to allow both of them to be lost.
(E) It is better to protect an endangered type of organism, even if doing so has some negative effects on another type of organism, than to do nothing at all.

The best answer is C.
先理解题意:wildflower会很快extinct如果不采取措施的话。但这个wildflower能与另外一种植物daisy cross-pollinate (异花受粉),从而produce viable seeds,产生一种杂交品种wildflower-daisy hybrid。虽然杂交后的植物differ markedly,但这是唯一一种阻止wildflower在其领域消失的方式。提问哪种原则能支持这种推理?
原文是cross-pollinate and produce seeds, 而不是preserve。
同上,preserve不对,而且vigorous没有提到。
“change a type” 与“produce seeds”、“wildflower-daisy hybrids”相符;“radical”与“markedly”相对应。 So C is the best answer.
“destroy”与“competing types”无关,排除。
negative effects 无关,排除
解题的关键是如何抽象原文,并运用无关排除法。(顺便把34题也说了:))
作者: capsicum    时间: 2011-7-5 17:50

34. Which one of the following is an assumption on which the conservationist’s reasoning depends?

A. The wildflower currently reproduces only by forming seeds.
B. The domesticated daisy was bred from wild plants that once grew in the wildflower’s range.
C. Increasing the population of the wildflower will also expand its range.
D. Wildflower-daisy hybrids will be able to reproduce.
E. The domesticated daisy will cross-pollinate with any daisy like plant.

思路:not+weaken。根据原命题成立则逆否命题成立的原理,把选项取非后,若能对文章产生削弱作用的,则为答案。
A. 与结论无关。另外也可以根据极端词only来进行排除。
B. 就算daisy不是in wildflower's range,wildflower-daisy hybrids也是in wildflower's range。
C. 无关。
D. 把选项取非,变成:wildflower-daisy hybrids不能再繁殖。这样的话,总有一天,随着wildflower的消失,wildflower-daisy hybrids也会消失的,这样就不能达到prevent total loss of the wildflower in its range的目的了,从而也对结论进行了削弱。因此D为答案。
E. 无关。

D is the best answer. 原文说“hybridization is the only means of preventing total loss of the wildflower in its range”, 如果hybrids不能reproduce,那wildflower还是会extinct。其他可在选项中加“not”看是否有影响结论。
作者: capsicum    时间: 2011-7-5 17:50

35. Because of increases in the price of oil and because of government policies promoting energy conservation, the use of oil to heat homes fell by 40 percent from 1970 to the present, and many homeowners switched to natural gas for heating. Because switching to natural gas involved investing in equipment, a significant switch back to oil in the near future is unlikely.

The prediction that ends the passage would be most seriously called into question if it were true that in the last few years.

A. the price of natural gas to heat homes has remained constant, while the cost of equipment to heat homes with natural gas has fallen sharply.
B. the price of home heating oil has remained constant, while the cost of equipment to heat home with natural gas has risen sharply.
C. the cost of equipment to heat homes with natural gas has fallen sharply, while the price of home heating oil has fallen to 1970 levels.
D. the cost of equipment to heat homes with oil has fallen sharply, while the price of heating with oil has fallen below the price of heating with natural gas
E. the use of oil to heat homes has continued to decline, while the price of heating oil has fallen to 1970 levels

思路:演绎题里,若结论中有比较的,则答案中也应该有比较。此处的结论中有“用oil好,还是用gas好”的比较,所以答案中必须有oil和gas的比较。

A. 只有gas和gas equipment的过去与现在的自身的比较,故不对。
B. 只有oil的过去与现在的自身的比较;gas equipment的过去与现在的自身比较。不对。
C. 与B的分析相同。
D. 有oil与gas的price的比较gas equipment的过去与现在的比较,说明了用oil比用gas便宜,而且也提到了oil equipment的造价急剧下降,所以用oil会比用gas好,因此switch back to oil是合算的,是likely的,对文章起到weaken的作用。
E. 只有oil自身的比较,不对。
作者: capsicum    时间: 2011-7-5 17:51

36. Sometimes when their trainer gives the hand signal for “Do something creative together” two dolphins circle a pool in tandem and then leap through the air simultaneously. On the other occasions, the same signal elicits synchronized backward swims or tail-waving. These behaviors are not simply learned responses to a given stimulus. Rather, dolphins are capable of higher cognitive functions that may include the use of language and forethought.

Which one of the following, if true, most strengthens the argument?

A. Mammals have some resemblance to one another with respect to bodily function and brain structure.
B. The dolphins often exhibit complex new responses to the hand signal.
C. The dolphins are given food incentives as part of their training.
D. Dolphins do not interact with humans the way they interact with one another.
E. Some of the behaviors mentioned are exhibited by dolphins in their natural habitat

思路:直接提出另一个事实,若此事实际能够说明结论成立的,则此事实为答案。

A. 没有提及cognitive functions,对结论无关。
B. 选项中的“new responses”,说明了dolphins能够经过思考,再作出一个新的反映,而不是单单的作出learned responses to a given stimulus. 此选项支持了结论中的cognitive functions,是答案。
C. 无关。
D. interact的方式不一定与cognition有关,也可能会与learned responses或条件反射有关。
E. 无关。
作者: capsicum    时间: 2011-7-5 17:51

Test 3

37. Editorialist: Drivers with a large number of demerit points who additionally have been convicted of a serious driving-relative offense should either be sentenced to jail or be forced to receive driver reeducation, since to do otherwise would be to allow a crime to go unpunished. Only if such drivers are likely to be made more responsible drivers should driver re-education be recommended for them. Unfortunately, it is always almost impossible to make drivers with a large number of demerit points more responsible drivers.
If the editorialist’s statements are true, they provide the most support for which one of the following?

A. Drivers with a large number of demerit points who have been convicted of a serious driving-related offense should be sent to jail.
B. Driver re-education offers the best chance of making drivers with a large number of demerit points responsible drivers.
C. Driver re-education is not a harsh enough punishment for anyone convicted of a serious driving-related offense who has also
D. Driver re-education should not be recommended for those who have committed no serious
E. Drivers with a larger number of demerit points but no conviction for a serious driving-related offense should receive driver re-education rather than jail

答案:A
思路:主编认为那些被罚了很多分并承认曾有过严重的驾驶违规的司机应该被判入狱或强制接受司机再教育,否则就是对该罪责的姑息。司机再教育只适合推荐给那些可能因此变得更有责任心的司机。不幸的是,那些被罚了很多分的司机总是不可能变得更有责任。因此,结论是-罚分高并有过严重驾驶违规的司机应该被送进监狱。
A: 答案符合原文推断。
B: 与原文的论述相反。
C: 原文中没有相关的论述。
D: 主编的论点中没有提到对没有严重驾驶违规的司机的处理意见。
E: 主编的论点中没有提到对没有严重驾驶违规的司机的处理意见。
作者: capsicum    时间: 2011-7-5 17:51

38. Amphibian populations are declining in numbers worldwide. Not coincidentally, the earth’s ozone layer has been continuously depleted throughout the last 50 years. Atmospheric ozone blocks UV-B, a type of ultraviolet radiation that is continuously produced by the sun, and which can damage genes. Because amphibians lack hair, hide, or feathers to shield them, they are particularly vulnerable to UV-B radiation. In addition, their gelatinous eggs lack the protection of leathery or hard shells. Thus, the primary cause of the declining amphibian population is the depletion of the ozone layer.

Each of the following, if true, would strengthen the argument EXCEPT:

A. Of the various types of radiation blocked by atmospheric ozone, UV-B is the only type that can damage genes.
B. Amphibian populations are declining far more rapidly than are the populations of non-amphibian species whose tissues and eggs have more natural protection from UV-B.
C. Atmospheric ozone has been significantly depleted above all the areas of the world in which amphibian populations are declining.
D. The natural habitat of amphibians has not become smaller over the past century.
E. Amphibian populations have declined continuously for the last 50 years.

答案:D
思路:原文论述臭氧层的破坏造成UV-B辐射过量是导致两栖动物数量减少的主要原因。
A:支持说明为什么是UV-B而不是其他原因。
B:支持说明为什么UV-B会降低两栖动物的数量。
C:提供空间论据说明臭氧层受损和两栖动物数量减少的相关性。
D:中性评价。不Weaken也不Strengthen.
E:提供时间论据说明臭氧层受损和两栖动物数量减少的相关性。

技巧:做EXCEPT题如果答案中出现原文没有的专有名词(地名和人名等)一般就选它。
作者: capsicum    时间: 2011-7-5 17:52

39. Quasars---celestial objects so far away that their light takes at least 500 million years to reach Earth---have been seen since 1963. For anything that far away to appear from Earth the way quasars do, it would have to burn steadily at a rate that produces more light than 90 billion suns would produce. But nothing that burns at a rate that produces that much light could exist for more than about 100 million years.

If the statements above are true, which one of the following must also be true on the basis of them?

A. Instruments in use before 1963 were not sensitive enough to permit quasars to be seen.
B. Light from quasars first began reaching Earth in 1963.
C. Anything that from Earth appears as bright as a quasar does must produce more light than would be produced by 90 billion suns.
D. Nothing that is as far from Earth as quasars are can continue to exist for more than about 100 million years.
E. No quasar that has ever been seen from Earth exists any longer.

答案:E
思路:
A:可能足够sensative,只是没有观察到而已。
B:同样只是没有观察到而已。
C:没有定义与地球的距离。
D:没有定义亮度
E:原文实际认为现在看到的来自Quasars的光是500万年前发出的。而在400万年前这些类星体就已经燃尽不存在了。
作者: capsicum    时间: 2011-7-5 17:52

40. More and more computer programs that provide solutions to mathematical problems in engineering are being produced, and it is thus increasingly unnecessary for practicing engineers to have thorough understanding of fundamental mathematical principles. Consequently, in training engineers who will work in industry, less emphasis should be placed on mathematical principles, so that space in the engineering curriculum will be available for other important subjects.

Which one of the following, if true, most seriously weakens the argument given for the recommendation above?

A. The effective use of computer program that provide solutions to mathematical problems in engineering requires an understanding of mathematical principles.
B. Many of the computer programs that provide solutions to mathematical problems in engineering are already in routine use.
C. Development of composites and other such new materials has meant that the curriculum for engineers who will work in industry must allow time for teaching the properties of these materials.
D. Most of the computer programs that provide solutions to mathematical problems in engineering can be run on the types of computers available to most engineering firms.
E. The engineering curriculum already requires that engineering students be familiar with and able to use a variety of computer programs.

Argument: 因为有了解决数学问题的电脑程序,所以工程师不用学数学了
A) Weaken, 因为数学是使用这些电脑程序的必要条件,B) C) D) E) irrelevant
作者: capsicum    时间: 2011-7-5 17:52

41. Raymond Burr played the role of lawyer Perry Mason on television. Burr’s death in 1993 prompted a prominent lawyer to say “Although not a lawyer, Mr. Burr strove for such authenticity that we feel as if we lost one of our own.” This comment from a prestigious attorney provides appalling evidence that, in the face of television, even some legal professionals are losing their ability to distinguish fiction from reality.

The reasoning in the argument is flawed because the argument

A. takes the views of one lawyer to represent the views of all lawyers
B. criticizes the lawyer rather than the lawyer’s statement
C. presumes that the lawyer is qualified to evaluate the performance of an actor
D. focuses on a famous actor’s portrayal of a lawyer rather than on the usual way in which lawyers are portrayed on television
E. ignores the part of the lawyer’s remark that indicates an awareness of the difference between reality and fiction

用排除法解决。因为A,B,C,D指出的错误原argument并没有犯。
作者: capsicum    时间: 2011-7-5 17:53

42. For next year, the Chef’s Union has requested a 10 percent salary increase for each of its members, whereas the Hotel Managers’ Union has requested only an 8 percent salary increase for each of its members. These facts demonstrate that the average dollar amount of the raises that the Chefs’ Union has requested for next year is greater than that of the raises requested by the Hotel Managers’ Union.

Which one of the following, if true, most strengthens the argument?

A. The Chefs’ Union has many more members than does the Hotel Managers’ Union.
B. The Chefs’ Union is a more powerful union than is the Hotel Managers’ Union and is therefore more likely to obtain the salary increases it requests
C. The current salaries of the members of the Chefs’ Union are, on average, higher than the current salaries of the members of the Hotel Managers’ Union
D. The average dollar amount of the raises that the members of the Chefs’ Union received last ear was equal to the average dollar amount of the raises that the members of the Hotel Managers’ Union received
E. The members of the Chefs’ Union received salary increases of 10 percent in each of the last two years, while the members of the Hotel Managers’ Union received salary increases of only 8 percent in each of the last two years

Percentage、基数和绝对值的关系
A. 无关
B. This is out of scope . The topic in the passage has nothing to do with the power of the 2 associations. The topic is only concerned with data analysis.
C. 基数大 (Percentage 大,基数又大,绝对值肯定大了)
D. 无关
E. 无关
作者: capsicum    时间: 2011-7-5 17:53

43. Zoos have served both as educational resources and as entertainment. Unfortunately, removing animals from their natural habitats to stock the earliest zoos reduced certain species’ populations, endangering their survival. Today most new zoo animals are obtained from captive breeding programs, and many zoos now maintain breeding stocks for continued propagation of various species. This makes possible efforts to reestablish endangered species in the wild.
Which one of the following statements is most strongly supported by the information above?
A. Zoos have played an essential role in educating the public about endangered species.
B. Some specimens of endangered species are born and bred in zoos.
C. No zoos exploit wild animals or endanger the survival of species.
D. Nearly all of the animals in zoos today were born in captivity.
E. The main purpose of zoos has shifted from entertainment to education.

参考答案:B
思路:
前提:指出动物园作为教育和娱乐两种目的,但是把动物从大自然装到笼子里减少了一些物种的数量,危及了他们的生存。
假设:现在大多数动物园的动物是通过俘获和饲养得到的,动物园通过此使稀有物种繁殖,从而使自然界的稀有物种能够有效的维持。

A. 文章没有提到,而且与所问的问题没太多相关性。
B. 正确。可以从文章中得到支持。
C. The sentence can not be reached by the passage, actually it is the opposite. From the passage we know that in the past certain species are reduced.
D. 说的是所有的动物,与文章新产生的动物most new zoo animals are obtained from captive breeding programs,范围不一致
E. 文章没有提到这一点。
作者: capsicum    时间: 2011-7-5 17:53

44. Legal theorist: It is unreasonable to incarcerate anyone for any other reason than that he or she is a serious threat to the property or lives of other people. The breaking of a law does not justify incarceration, for lawbreaking proceeds either from ignorance of the law or of the effects of one’s actions, or from the free choice of the lawbreaker. Obviously mere ignorance cannot justify incarcerating a lawbreaker, and even free choice on the part of the lawbreaker fails to justify incarceration, for free choice proceeds from the desires of an agent, and the desires of an agent are products of genetics and environmental conditioning, neither of which is controlled by the agent
The claim in the first sentence of the passage plays which one of the following roles in the argument
A. It is offered as a premise that helps to show that no actions are under the control of the agent
B. It is offered as background information necessary to understand the argument
C. It is offered as the main conclusion that the argument is designed to establish
D. It is offered as evidence for the stated claim that protection of life and property is more important than retribution for past illegal acts
E. It is offered as evidence for the stated claim that lawbreaking proceeds from either ignorance of the law, or ignorance of the effects of one’s actions, or free choice

参考答案:C
思路:第一句指出出了严重危害他人生命和财产之外,其它违反法律的事件而监禁是不合理的结论,而后面的句子结构给出了相应的解释和说明。
A:第一句诗结论不是假设前提。
B:背景信息也不正确,应该是结论。
C:正确,is is offered as the main conclusion.
D:the evidences offered for the argument are stated behind.
E:错误,见D。
作者: capsicum    时间: 2011-7-5 17:53

45. Commissioner: Budget forecasters project a revenue shortfall of a billion dollars in the coming fiscal year. Since there is no feasible way to increase the available funds, our only choice is to decrease expenditures. The plan before you outlines feasible cuts that would yield savings of a billion dollars over the coming fiscal year. We will be able to solve the problem we face, therefore, only if we adopt this plan.
This reasoning in the commissioner’s argument is flawed because this argument
A. relies on information that is far from certain
B. confuses being an adequate solution with being a required solution
C. inappropriately relies on the opinions of experts
D. inappropriately employs language that is vague
E. takes for granted that there is no way to increase available funds

参考答案:B
思路:年度财务预算下一财年收入不足量one billion dollars,由于没有feasible way增加可得资金,只有削减开支。因而只是做了一个如何之间节省10亿美金的计划,it is just a required solution but not adequate maybe.

A:信息不准,错误。
B:正确,见思路分析。
C:不正确的依赖专家观点,无法得出。
D:思路模糊,文章思路是比较的清晰的。
E:前提已经指出没有feasible way,所以E错误。
作者: capsicum    时间: 2011-7-5 17:54

46. Twelve healthy volunteers with the Apo-A-IV-1 gene and twelve healthy volunteers who instead have the Apo-A-IV-2 gene each consumed a standard diet supplemented daily by a high-cholesterol food. A high level of cholesterol in the blood is associated with an increased risk of heart disease. After three weeks, the blood cholesterol levels of the subjects in the second group were unchanged, whereas the blood cholesterol levels of those with the Apo-A-IV-1 gene rose 20 percent.

Which one of the following is most strongly supported by the information above?

A. Approximately half the population carries a gene that lowers cholesterol levels.
B. Most of those at risk of heart disease may be able to reduce their risk by adopting a low-cholesterol diet.
C. The bodies of those who have the Apo-A-IV-2 gene excrete cholesterol when blood cholesterol reaches a certain level.
D. The presence of the Apo-A-IV-1 gene seems to indicate that a person has a lower risk of heart disease.
E. The presence of the Apo-A-IV-2 gene may inhibit the elevation of blood cholesterol.

参考思路:试验题类型。用同一种方法对两种事物进行研究,通常考察两种事物对这种方法是否有应激性,是同向还是反向。

A:无关
B:无关
C:与所提供的信息相反
D:无关,
E:正确,inhibit the elevation of blood cholesterol-----〉the blood cholesterol levels of the subjects in the second group were unchanged
作者: capsicum    时间: 2011-7-5 17:54

47. High school students who feel that they are not succeeding in school often drop out before graduating and go to work. Last year, however, the city’s high school dropout rate was significantly lower than the previous year’s rate. This is encouraging evidence that the program instituted two years ago to improve the morale of high school students has begun to take effect to reduce dropouts.

Which one of the following, if true about the last year, most seriously weakens the argument?

A. There was a recession that caused a high level of unemployment in the city.
B. The morale of students who dropped out of high school had been low even before they reached high school.
C. As in the preceding year, more high school students remained in school than dropped out.
D. High schools in the city established placement offices to assist their graduates in obtaining employment.
E. The antidropout program was primarily aimed at improving students’ morale in those high schools with the highest dropout rates.

The passage puts forward a phenomenon that last year, however, the city’s high school dropout rate was significantly lower than the previous year’s rate; the passage attributes the phenomenon to the explanation that there is some program and the program helps the students’ morale. However if there are other reasons that could explain that mentioned phenomenon, the argument will be seriously weakened.
Choice A, by pointing out that a recession caused a high level of unemployment in the city, explains the phenomenon clearly. Furthermore, Choice A negates the effects of the so-called morale program. The students stay in the school not because they want to, but because they cannot find jobs so they have to stay.
Choice B has nothing to do with the reasoning in the passage because the passage does not deal with anything about the students before they join high school.
Choice C deals with the students remain in high school. However the passage discusses about students who dropped out.
Choice D clarifies what the program did about helping the students. This does not weaken the argument in the passage.
Choice E, just as D, only tells about the content of the program, does not weaken the argument.

看看这句话High school students who feel that they are not succeeding in school often drop out before graduating and go to work.这里是说学生是因为觉得不可能在学校取得成功才辍学找工作的。仅仅是帮助学生找工作不能解决学生的moral问题。因此D没有在根本上削弱。---- 深海的小美人鱼

因为学校成立了 placement offices 来帮助毕业生找工作,所以学生就不退学了
这句话的前提是学校帮不帮忙找工作决定学生退不退学,这在原文中并没有说明。如果单从学生不退学讲,这可以算作是支持。而原文的“drop out before graduating and go to work”的必然前提是学生可以go to work,也就是有地方可去,所以否定这个前提学生就不会退学,因此构成了“就业形势不好,学生就不退学了”这个他因。--- seanql
作者: capsicum    时间: 2011-7-5 17:54

48. The television show Henry was not widely watched until it was scheduled for Tuesday evenings immediately after That’s Life, the most popular show on television. During the year after the move, Henry was consistently one of the ten most-watched shows on television. Since Henry’s recent move to Wednesday evenings, however, it has been watched by far fewer people. We must conclude that Henry was widely watched before the move to Wednesday evenings because it followed That’s Life and not because people especially liked it.

Which one of the following, if true, most strengthens the argument?

A. Henry has been on the air for three years, but That’s Life has been on the air for only two years.
B. The show that replaced Henry on Tuesdays has persistently had a low number of viewers in the Tuesday time slot.
C. The show that now follows That’s Life on Tuesdays has double the number of viewers it had before being moved.
D. After its recent move to Wednesday, Henry was aired at the same time as the second most popular show on television.
E. That’s Life was not widely watched during the first year it was aired.

答案:C
思路:直接举证说明原因
A:无关
B:无关
C:直接举证说明原因,紧跟That’s Life的节目观众数量翻番
D:削弱
E:无关
作者: capsicum    时间: 2011-7-5 17:54

Questions 49-50
Joseph: My encyclopedia says that the mathematician Pierre de Fermat died in 1665 without leaving behind any written proof for a theorem that he claimed nonetheless to have proved. Probably this alleged theorem simply cannot be proved, since---as the article points out---no one else has been able to prove it. Therefore it is likely that Fermat was either lying or else mistaken when he made his claim.
Laura: Your encyclopedia is out of date. Recently someone has in fact proved Fermat’s theorem. And since the theorem is provable, your claim---that Fermat was lying or mistaken---clearly is wrong.

49. Joseph’s statement that “this alleged theorem simply cannot be proved” plays which one of the following roles in his argument?

A. an assumption for which no support is offered
B. a subsidiary conclusion on which his argument’s main conclusion is based
C. a potential objection that his argument anticipates and attempts to answer before it is raised
D. the principle claim that his argument is structured to refute
E. background information that neither supports nor undermines his argument’s conclusion

答案:B
思路:
Joseph的论证过程
EVIDENCE1:百科全书说:P宣称已证明某理论,但没有留下任何书面证据。(My encyclopedia says …….have proved.)
EVIDENCE2:没有人能够证明(since…. prove it)
Mid Conclusion:P没有证明他的理论。(EVIDENCE支持得到的中间结论)
Conclusion:P说谎或他的CLAIM错。(由EVIDENCE1和Mid Conclusion得到)
作者: capsicum    时间: 2011-7-5 17:55

50. Which one of the following most accurately describes a reasoning error in Laura’s argument?

A. It purports to establish its conclusion by making a claim that, if true, would actually contradict that conclusion.
B. It mistakenly assumes that the quality of a person’s character can legitimately be taken to guarantee the accuracy of the claims that person has made.
C. It mistakes something that is necessary for its conclusion to follow for something that ensures that the conclusion follows.
D. It uses the term “provable” without defining it.
E. It fails to distinguish between a true claim that has mistakenly between believed to be false and a false claim that has mistakenly been believed to be true.

答案:C
思路:
Laura的论证过程:该理论已证明—>P没有说谎,他的CLAIM对。理论对并不能表明是P证明的,这只是必要条件,不是充分条件。
作者: capsicum    时间: 2011-7-5 17:55

51. People who have political power tend to see new technologies as a means of extending or protecting their power, whereas they generally see new ethical arguments and ideas as a threat to it. Therefore, technical ingenuity usually brings benefits to those who have this ingenuity, whereas ethical inventiveness brings only pain to those who have this inventiveness.

Which one of the following statements, if true, most strengthens the argument?

A. Those who offer new ways of justifying current political power often reap the benefits of their own innovations.
B. Politically powerful people tend to reward those who they believe are useful to them and to punish those who they believe are a threat.
C. Ethical inventiveness and technical ingenuity are never possessed by the same individuals
D. New technologies are often used by people who strive to defeat those who currently have political power.
E. Many people who possess ethical inventiveness conceal their novel ethical arguments for fear of retribution by the politically powerful.

B is correct. The passage reaches the conclusion that technical ingenuity usually brings benefits to those who have this ingenuity, whereas ethical inventiveness brings only pain to those who have this inventiveness. However the only reason is that People who have political power tend to see new technologies as a means of extending or protecting their power, whereas they generally see new ethical arguments and ideas as a threat to it. From the reason to the conclusion, there is a jump of concept. We need a link for the 2 concepts: people who have political power and the ingenuity. Choice B, by establishing the link, provides the strongest support. Choice B is actually an assumption for the argument.
A, C does not bear on the reasoning of the passage. D weakens the argument. E does not concern, because what many people do has nothing to do with the conclusion.

52. Birds need so much food energy to maintain their body temperatures that some of them spend most of their time eating. But a comparison of a bird of a seed-eating species to a bird of a nectar-eating species that has the same overall energy requirement would surely show that the seed-eating bird spends more time eating than does the nectar-eating bird, since a given amount of nectar provides more energy than does the same amount of seeds

The argument relies on which one of the following questionable assumptions

A. Birds of different species do not generally have the same overall energy requirements as each other
B. The nectar-eating bird does not sometimes also eat seeds
C. The time it takes for the nectar-eating bird to eat a given amount of nectar is not longer than the time it takes the seed-eating bird to eat the same amount of seeds
D. The seed-eating bird does not have a lower body temperature than that of the nectar-eating bird
E. The overall energy requirements of a given bird do not depend on factors such as the size of the bird, its nest-building habits; and the climate of the region in which it lives

这就是一个排除它因型假设呀!为什么一定要去反呢在去看呢,好像这里好多网友都在这么做。 即使取反,那肯定削弱了原来命题:它因存在,不是吃得多,而是吃得慢造成的时间花费差异。--- dorbear

Nectar鸟和seed鸟需要能量一样多,但nectar食物单个能量大,所以nectar鸟吃的数量少。结论:nectar鸟吃东西花的时间少。 使以上必然成立的条件(assumption)是nectar鸟一定吃的更快,不然还是有可能花的时间比seed鸟多。和C相反,所以c questionable。---- seanql
作者: capsicum    时间: 2011-7-5 17:55

53. When investigators discovered that the director of a local charity had repeatedly overstated the number of people his charity had helped, the director accepted responsibility for the deception. However, the investigators claims that journalists were as much to blame as the director was for inflating the charity’s reputation, since they had naively accepted what the director told them, and simply reported as fact the numbers he gave them.

Which one of the following principles, if valid, most helps to justify the investigators’ claim

A. Anyone who works for a charitable organization is obliged to be completely honest about the activities of that organization.
B. Anyone who knowingly aids a liar by trying to conceal the truth from others is also a liar.
C. Anyone who presents as factual a story that turns out to be untrue without first attempting to verify that story is no less responsible for the consequences of that story than anyone else is.
D. Anyone who lies in order to advance his or her own career is more deserving of blame than someone who lies in order to promote a good cause.
E. Anyone who accepts responsibility for a wrongful act that he or she committed is less deserving of blame than someone who tries to conceal his or her own wrongdoing.

同51题:

参考思路:加强型题型。

对作者的论述,进行简单归纳形成加强。Anyone who presents as factual a story that turns out to be untrue without first attempting to verify that story is no less responsible for the consequences of that story than anyone else is.
作者: capsicum    时间: 2011-7-5 17:56

54. Telephone companies are promoting “voice mail” as an alternative to the answering machine. By recording messages from callers when a subscriber does not have access to his or her telephone, voice mail provides a service similar to that of an answering machine. The companies promoting this service argue that it will soon make answering machines obsolete, since it is much more convenient, more flexible, and less expensive than an answering machine.

Which one of the following, if true, most calls into question the argument made by the companies promoting voice mail?

A. Unlike calls made to owners of answering machines, all telephone calls made to voice-mail subscribers are completed, even if the line called is in use at the time of the call.
B. The surge in sales of answering machines occurred shortly after they were first introduced to the electronics market.
C. Once a telephone customer decides to subscribe to voice mail, that customer can cancel the service at any time.
D. Answering machines enable the customer to hear who is calling before the customer decides whether to answer the telephone, a service voice mail does not provide.
E. The number of messages a telephone answering machine can record is limited by the length of the magnetic tape on which calls are recorded

答案: D
思路: 段落论述X方式(voice mail)在某些方面的优点大于Y方式(answering machine), 将会导致Y的过时, 此时weaken的方向在于找出X劣于Y的缺点,或Y优于X的优点.
A. 提到voice mail的优点,对段落支持.
B. 无关选项,answering machine的销售情况与段落无关
C. 无关选项, 客户可在任何时间取消voice mail, 无X,Y的优劣比较
D. 正确, 提出了answering machine优于voice mail的优点.
E. 提出answering machine的缺点,支持了段落.
作者: capsicum    时间: 2011-7-5 17:56

Questions 55-56
The simple facts are these: the number of people killed each year by bears is about the same as the number of people killed by lightning of golf courses. And the number of people killed by lightning on golf courses each year is about the same as the number of people electrocuted by electric blenders. All the horrible myths and gruesome stories aside, therefore a grizzly bear is in fact about as dangerous as an electric blender or a game of golf.

55. Which one of the following is an assumption that the author relies upon in the passage?

A. Most incidents involving grizzly bears are fatal.
B. Grizzly bears are no longer the danger they once were.
C. The number of fatalities per year is an adequate indication of something’s dangerousness.
D. A golf course is a particularly dangerous place to be in a thunderstorm.
E. Something is dangerous only if it results in death in the majority of cases.

答案: C
思路: 搭桥法, 建立A与B的联系. 文章由三种情况导致people的死亡数量相同的事实推理到此三种情况的危险度相同,此时要找出死亡数量与危险度的关系.
A, B, D分别讲述了grizzly bear的伤害是致命的, grizzly bear 不再危险, golf course是雷击的特殊情况, 没有提及死亡数量与危险度的关系, 属无关选项.
E强调只有在大多数cases中出现死亡才为危险, 与A, B比较无关,
只有C是正确的,建立了A,B的关系, 死亡数量可准确地说明危险度.
作者: capsicum    时间: 2011-7-5 17:56

56. Which one of the following, if true, would most effectively undermine the author’s argument?

A. Although the number of people killed by lightning on golf courses each year is very small, the total number of lightning fatalities is many times greater.
B. Electric blenders are among the safest housed hold appliances; were the author to compare fatalities from electrical appliances in general, she would get a much higher figure.
C. Most people would rather take their chances with benders and golf games than with grizzly bears.
D. Bears in general---including black, brown, and cinnamon bears, as well as grizzly bears---kill many more people than do electric blenders.
E. Statistics show that the number of times people use electric blenders each year exceeds the number of times people play golf each year, which in turn far exceeds the number of contacts people have with grizzly bears each year.

答案: E
思路: 否定死亡数量与危险度的关系,找出影响危险度的另一个因素.
A: 无关选项, 被电击致死的总数量与因lightning on golf courses致死的数量比较, 属无关比较.
B: 无关选项, 其他electrical appliance的死亡数量更高, 与比较无关.
C: 大多数人的态度, 无关选项
D: 熊比electric blender杀的人多, 无关比较.
E: 正确, 人们接触某种情况的时间 (机会) 越多 (大), 其危险度越高.
作者: capsicum    时间: 2011-7-5 17:56

57. Free public education is the best form of education there is. Therefore, we must fight to ensure its continued existence; that is, we must be ready to defend the principle of equality of educational opportunity. Because this principle is well worth defending, it is clear that free public education is better than any other form of education.

Which one of the following illustrates the same weak reasoning as found in the passage?

A. I love music, and that’s why I listen to it constantly. I have my stereo or radio on every waking minute. Since I lay music all the time, I must really love it.
B. Books are my most valuable possessions. My books are like my friends---each pleases me in different ways. Just as I would give up everything to save my friends, so too with my books.
C. I would much rather be poor and respected than be rich and despises. To have the respect of others is far more valuable than to have millions of dollars.
D. I have never been betrayed by any of my friends. They have been true to me through good times and bad. Therefore I will never betray any of my friends.
E. Because every plant I have ever seen has green leaves, I have concluded that all plants must have green leaves. This looks like a plant but it does not have green leaves, so it cannot be a plant.

A. 选项中,因为爱音乐总开着音响;又以开音响为事实,证明自己爱音乐;这和原题中的逻辑错误相似。原题中,因为公共教育好所以要维护该项制度;又以努力维护制度为事实,证明公共教育好。The fallacy in the passage is circle——自说自话。The best answer is A.
B. 选项逻辑上没有什么错误吧。
C. 观点虽然有些偏激,但逻辑错误与原文并不相同。
D. 以事实为基础,感恩图报,没什么错的。
E. 因为见到过的星球都有绿色叶子,得出因为没有绿叶,虽然看起来象星球也不是星球,推出荒谬的结论。其中的逻辑错误应该是以偏概全吧,这个选项迷惑性满大的。
作者: capsicum    时间: 2011-7-5 17:57

58. Some people say that the scarcity of food is a function of the finite limits of the earth’s resources, coupled with a relentless rate of population growth. This analysis fails to recognize, however, that much of the world’s agricultural resources are used to feed livestock instead of people. In the United States, for example, almost one-half of the agricultural acreage is devoted to crops fed to livestock. A steer reduces twenty-one pounds of inexpensive grain to one pound of expensive meat. Thus, the scarcity of food is not merely a function of limited resources and population growth.

Which one of the following is an assumption that would allow the conclusion in the argument to be properly drawn?

A. People prefer eating meat to eating grain.
B. Meat is twenty-one times more expensive than grain.
C. The limits of the earth’s agricultural resources are not finite.
D. More than one-half of the agricultural acreage in the United States is devoted to drops fed to humans.
E. Growing crops for human consumption on the acreage currently devoted to crops for livestock will yield more food for more people.

E is the best answer. 喂养牲畜消耗的粮食为人们创造更多的食物。(题目内容是:有观点认为食物缺乏是因为土地资源有限而人口数量却不断增多。但作者认为该观点忽视了农产品中有相当大一部分是用来喂家畜的。在美国几乎有一半土地的粮食是为了养家畜。1磅肉要用去21磅廉价的谷物。因此,食物缺乏不仅是因为土地资源有限而人口数量却不断增多。)根据题目要求,我们只须反驳养家畜对粮食紧缺造成的影响。选项A中即使人们爱吃肉只会引起更多的粮食消耗,不能反驳作者; B并不能充分改变人们的取向;C强词夺理;D重复事实。
作者: capsicum    时间: 2011-7-5 17:57

59. Top college graduates are having more difficulty demonstrating their superiority to prospective employers than did the top students of twenty years ago when an honors degree was distinction enough. Today’s employers are less impressed with the honors degree. Twenty years ago no more than 10 percent of a given class graduated with honors. Today, however, because of grade inflation, the honors degree goes to more than 50 percent of a graduating class. Therefore, to restore confidence in the degrees they award, colleges must take steps to control grade inflation.

Which one of the following is an assumption that, if true, would support the conclusion in the passage?

A. Today’s students are not higher achievers than the students of twenty years ago.
B. Awarding too many honors degrees causes colleges to inflate grades.
C. Today’s employers rely on honors ranking in making their hiring decisions.
D. It is not easy for students with low grades to obtain jobs.
E. Colleges must make employers aware of the criteria used to determine who receives an honor degree.

The best answer is A. 如果现在的优秀学生数量比20年前多,那么授予更多的honor degree就很正常了,也许除了荣誉学位,学生应该用别的方面来证明自己。所以只有当现在的学生素质不值得被授予象20年前那样的荣誉学位的时候,整个论证才有道理。
作者: polo_xiong    时间: 2011-7-8 09:54

60. One of the more reliable methods of determining regional climatic conditions in prehistoric periods is to examine plant pollen trapped in glacial ice during ancient times. By comparing such pollen samples with spores take from modern vegetation, scientists can figure out approximately what the weather was like at the time of pollen deposition. Furthermore, by submitting the prehistoric samples to radiocarbon dating techniques, we can also determine when certain climatic conditions were prevalent in that portion of the globe.

Which one of the following may be inferred from the information in the passage?

A. The earth has undergone several glacial periods.
B. Radiocarbon dating can be corroborated by glacial evidence.
C. Similarities between prehistoric and contemporary climates do not exist.
D. Pollen deposition is a fairly continuous process.
E. Certain flora are reliably associated with particular climatic conditions.

INFERENCE question is very similar to ASSUMPTION in that both are required conditions in the argument. So inference is not a conclusion itself of the argument. It is one condition that must be true for the conclusion to hold. The line of reasoning can be as follows:
Argument: A à B. Ask for inference C. The links are A à B à C. Or Not C à Not B. A and B are the premise and conclusion in the argument respectively. C is the inference.
No. 60 is a good example. What the passage says is that scientists can figure out the weather condition in ancient times by examine the pollen from those times. So the hidden premise is that there is a connection between groups of plants and climatic conditions. If E is not true, and there is no connection between plants and climatic conditions, the statement that scientists can find out the climatic conditions by studying pollen can not hold.
So, for INFERENCE questions, you should treat it the same way as ASSUMTION question. First, understand the conclusion, or the statement in this question. And find the REQUIRED condition in the choices. To test the answer, use “NOT” and put it back in the argument to see if conclusion still holds.
Let’s take a look at the rest of the choices.
A: Irrelevant. Scientist can use pollen to find out the climatic conditions no matter how many times the earth has undergone glacial periods. When you see a choice with words expressing “degree” or “times” like rarely, most, many, less, several, probably, very, none, etc., pay attention to what the passage says. You need to make sure that these words can be inferred from the passage.
B. Irrelevant. The argument says that using radiocarbon on the samples can get further information. What B says is that glacial evidence supports radiocarbon dating. In another words, glacial evidence proves what radiocarbon dating finds out. It is not hinted by the argument or required for the conclusion.
C. Irrelevant to the statement. Use “not” to test.
D. Looks like an answer. But the statement is not about the continuity of climatic conditions. So D is irrelevant.
作者: capsicum    时间: 2011-7-8 09:56

61. Investigators concluded that human failure was not responsible for the fatal airplane crash last August, and since that time new and more stringent rules for identifying and reporting mechanical problems have been in effect. That accounts for the fact that reports of airplane mechanical problems have increased in frequency by 50 percent since last August.

Which one of the following is an assumption underlying the argument in the passage?

A. Airplane travel is still relatively safe, despite the increase in reported mechanical problems.
B. Mechanical problems in airplanes have increased dramatically since last August.
C. Mechanical problems in airplanes have not increased by 50 percent since last August.
D. Airlines are less reluctant to report mechanical problems than they previously were.
E. Mechanical problems in airplanes have become easier to detect since last August.

A. 文中第一句the fatal airplane crash last August已经客观存在,说明飞机旅行并非相当安全。与文中事实相反。
B. 用于识别和报告机械故障问题的更加严格的新规定已经生效,这才是the fact that reports of airplane mechanical problems have increased in frequency by 50 percent since last August的真正原因。而B选项Mechanical problems in airplanes是它因,削弱结论。
C. 正确。“Mechanical problems in airplanes have increased by 50 percent since last August”是用他因来削弱结论,加上NOT,则是排除他因的假设。
D. 诉诸于航空公司的主观意愿,与过去相比较,都跟现在的情况无关。
E. become easier to detect无关词。
作者: capsicum    时间: 2011-7-8 09:56

62. Brushing your teeth regularly no matter which toothpaste you use, will reduce your chances of tooth decay. Scientists have concluded that, when you brush, you reduce tooth decay by removing the film of plaque that forms on teeth and gums. So, you can forget about fluorides: brush your teeth carefully and say goodbye to cavities.

Which one of the following is a criticism of the reasoning in the argument?

A. Brushing with fluoride toothpaste has been shown to reduce tooth decay.
B. The fact that brushing will reduce tooth decay does not show that fluorides are of no value.
C. Few people adequately remove plaque by brushing.
D. People have plaque on their teeth most of the time.
E. Scientists have been wrong about fluorides.

结论——你能忘掉氟化物:请认真地刷牙才会告别牙洞。取非:氟化物不能被忘掉,不是只刷牙就能远离牙洞。

A. 结论中显示了作者对于两者的态度,否定氟化物,肯定刷牙。削弱则必须肯定氟化物,把不同类的两者放在一起,因果的关系不明显,容易混淆。
B. 正确。刷牙会减少牙齿腐烂的事实并不能表明氟化物就没有价值,即,不能因为一样东西有作用就说另外一个东西没作用:准确无误指出了原文论证的漏洞。
C. 诉诸于一般人的情况,无关。
D. Most of the time无关词。问题同C
E. 诉诸于权威,无关。
作者: capsicum    时间: 2011-7-8 09:57

63. Some good cooks are gourmet cooks who pride themselves on always using extravagantly rich ingredients in elaborate recipes. Some good cooks can be characterized as fast-food cooks. They may use rich ingredients as long as the recipes are easy to follow and take little time. Other good cooks are health food enthusiasts, who are concerned primarily with the nutritional value of food. But even though not all good cooks are big eaters, they all enjoy preparing and serving food.

If the information in the passage is true, which one of the following CANNOT be true?

A. Most good cooks do not use extravagantly rich ingredients.
B. Everyone who enjoys preparing and serving food is a good cook.
C. More good cooks who use extravagantly rich ingredients are big eaters than are good cooks who do not use such ingredients.
D. There are fewer good cooks who enjoy serving and preparing food than there are good cooks who are big eaters.
E. Gourmet cooks, fast-food cooks, and cooks who are health food enthusiasts are all big eaters.

A. 原文的第一句,从原文推出。
B. 从结论可以推出。
C. 推不出这种比较,无关。
D. 正确。结论:但是即使并非所有的好厨师是big eaters,他们却全都喜欢准备和供应食物。说明喜欢准备和供应食物的好厨师的人数更多,是big eaters的好厨师的人数相比较少。D选项与原文的结论相反。
E. 从原文推出。
作者: capsicum    时间: 2011-7-8 09:57

Questions 64-65
If the city council institutes new parking regulations, city revenues will surely increase, since studies have conclusively shown that, if such parking regulations are put into effect, there is an increase in parking violations, and an increase in parking violations will result in a greater number of parking fines collected.

64. If the statements in the passage are true, which one of the following must also be true?

A. Unless there is an increase in the number of parking violations in the city, city revenues will not increase.
B. If the city council institutes new parking regulations, the council will fall from favor with the citizens.
C. The city council will institute new parking regulations only if an increase in city revenues can be expected to result.
D. If the city council’s new regulations cause more parking violators to be ticketed, the city revenues will increase.
E. Unless the city institutes a complex system of parking regulations, the city cannot expect traffic violations to increase.

Choice C just repeats the conclusion: institution of new regulations -> increase of city revenues. it fails to fill the gap between institution of regulations and increase of revenues.
new regulations -> more violations-> more fines collected(increase of revenues)

D is correct.
作者: capsicum    时间: 2011-7-8 09:57

65. According to advertisements, the higher a suntan lotion’s sun protection factor, or SPE, the more protection from sunburn. In order for a suntan lotion to work, however, one has to remember to put it on before going in the sun, put on an adequate amount to cover the skin, and reapply it as needed. Therefore, it really does not matter what SPE a suntan lotion has.

Which one of the following best identifies the error in reasoning made in the passage?

A. It is unreasonable to assume that the only purpose of a suntan lotion is to provide protection from sunburn.
B. Because some people get sunburned more easily than other, the fact that there are different SPEs cannot be ignored.
C. It cannot be concluded that the SPEs is not important just because there are requirements for the application of the suntan lotion.
D. It is unreasonable to assume that all suntan lotions required the same application procedures in order to work effectively.
E. There is no reason to assume that manufacturers are unaware that people sometimes forget to apply suntan lotion before going in the sun.

This is exactly the same as No 62. 只有C明确提出不能因为一样东西有作用就说另外一个东西没作用:准确无误指出了原文论证的漏洞。
A, The passage has nothing to do with the “only purpose”.
B, Because the passage has already make clear that people can apply many times of the lotion for difference people, so the idea in choice B is not a lope hole in the reasoning.
C,By pointing out that it is not right to eliminate the effectiveness of SPEs because there are other requirements.
D,There is no assumption in the passage that all suntan lotions required the same application procedures.
E,Anything about manufactures does not bear on the reasoning in the passage.
作者: capsicum    时间: 2011-7-8 09:58

66. A recent survey showed that many workers in a certain company are dissatisfied with their jobs. The survey also showed that most of the dissatisfied workers believe that they have little control over their job assignments. Therefore, to increase workers’ job satisfaction the company’s management need only concentrated on changing workers’ beliefs regarding the degree of control they have over their job assignment.

Which one of the following, if also shown by the survey, would most seriously call into question the conclusion made by the author of the passage?

A. The dissatisfied workers feel that their wages are too low and working conditions are unsatisfactory.
B. The number of workers in the company who are satisfied with their jobs is greater than the number of workers who are dissatisfied.
C. The workers in the company are more dissatisfied than workers in other companies.
D. Most people in company management believe that the workers already have too much control over their work.
E. The workers in the company who are satisfied with their jobs believe that they have a lot of control over their job assignments.

他因法进行削弱。A is correct.
(B) The number of workers in the company who are satisfied with their jobs is greater than the number of workers who are dissatisfied. 无关数字
(C) The workers in the company are more dissatisfied than workers in other companies. 无关对比
(D) Most people in company management believe that the workers already have too much control over their work. 无关比较
(E) The workers in the company who are satisfied with their jobs believe that they have a lot of control over their job assignments. 支持
作者: capsicum    时间: 2011-7-8 09:58

67. It has always been difficult to understand the basis of politics in the People’s Republic of China. Because the system is effectively closed, it is impossible to know with any degree of confidence who is allied with whom and for what reasons. Yet Chinese politics does exhibit many of the external characteristics of factional political systems, as found in more open societies. It is legitimate to conclude, therefore, that China has a factional political system.

Which one of the following, if true, would confirm the author’s conclusion that China has a factional political system?

A. All open political systems are factional political systems.
B. All factional political systems are closed political systems.
C. All closed political systems are factional political systems.
D. China’s political system is more open than many existing factional political systems.
E. China’s political system is more closed than all existing factional political systems.

答案:C

思路:加强题。前提1,(A)中国政治系统——封闭;前提2,中国政治——派系政治系统;结论,(B)中国——派系政治系统。加强结论。

A. open political systems 文中未提及,与open societies无关,不要混淆。
B. B是A,AB之间架桥,但起到加强前提的作用,与问题要求相反。
C. A是B,AB之间架桥,起到加强结论的作用,
D. 架桥方向错误,open与否与结论无关。
E. 同D。
作者: capsicum    时间: 2011-7-8 10:08

69. History textbooks frequently need to be revised. The reasons for this are clear: new discoveries of documents and remains, the discovery of mistaken inferences in prior histories, the discovery of previously unnoticed relationships among data, and the application of hitherto undiscovered principles of natural science all may indicate inadequacies in current history texts. Any of these considerations may require that the past be reinterpreted in a manner that is new and more illuminating.

Which one of the following can be inferred from the argument in the passage?

A. The interpretation of historical events is affected by natural science.
B. The past is constantly renewed because of illuminating reinterpretations.
C. History books are outdated as soon as they are written.
D. Natural scientists also function as historians.
E. Historians’ mistaken inferences are caused by unnoticed relationships among data.

答案:A
思路:归纳题,正确选项从原文推出,范围比原文窄。可用无关词排除法。
A. 文中提到,未经发现的自然科学法则会造成历史课本的不充分,需要被重新解释。说明历史事件的解释是受自然科学影响的。正确。
B. 混淆选项。根据最后一个句子,可知原文的意思是过去以一种新的且更加明确的方式被重新解释,而不是因为明确的解释而被经常更新。
C. 根据头脑空白原则,不能由历史课本需要经常被修正就推出历史课本写的时候就过时了。
D. 文中未出现诸如自然科学家、历史学家,无关词排除。
E. 同D。
作者: capsicum    时间: 2011-7-8 10:08

70. A low-pressure weather system is approaching Plainville; rainfall results from about 70 percent of such systems in the Plainville area. Moreover, the current season, spring, is the time of year in which thundershowers, which sometimes result from low-pressure systems, are most likely to occur in Plainville.

Knowing which one of the following, in addition to the information above, would be most useful for determine the probability that Plainville will have a thundershower soon?

A. the percentage of thundershowers in Plainville that occur in the spring
B. the percentage of spring rainfalls in Plainville that are thundershowers
C. the percentage of thundershowers in Plainville that result from low-pressure systems
D. whether low-pressure systems in other areas are associated with rainfall
E. whether Plainville has more or fewer thundershowers each spring than do near by towns

参考答案:C
思路:文中给出两个条件,1是低压天气将到达,而70%的低压天气导致降雨,2是现在正是1年中雷阵雨最多的季节且雷阵雨有时也是由于低压天气导致;答案(C)给出了低压天气导致雷阵雨的比率,用70%乘以这个比率,就可以得到下雷阵雨的概率了。
A、B没能和低压天气联系起来,D、E为无关选项
作者: capsicum    时间: 2011-7-8 10:09

71. More than a year ago, the city announced that police would crack down on illegally parked cars and that resources would be diverted from writing speeding tickets to ticketing illegally parked cars. But no crackdown has taken place. The police chief claims that resources have had to be diverted from writing speeding tickets to combating the city’s staggering drug problem. Yet the police are still writing as many speeding tickets as ever. Therefore, the excuse about resources being tied up in fighting drug-related crime simply is not true.

The conclusion in the passage depends on the assumption that

A. every member of the police force is qualified to work on combating the city’s drug problem
B. drug-related crime is not as serious a problem for the city as the police chief claims it is
C. writing speeding tickets should be as important a priority for the city as combating drug-related crime
D. the police could be cracking down on illegally parked cars and combating the drug problem without having to reduce writing speeding tickets
E. the police cannot continue writing as many speeding tickets as ever while diverting resources to combating drug-related crime

参考答案:E
思路:结论是由于对超速行驶开出的罚单并未减少,所以资源并未转移,其逆否命题则是,资源如果真的转移,罚单一定减少,也就是E指出的 — 罚单不可能与原来一样多。
作者: capsicum    时间: 2011-7-8 10:09

72. A gas tax of one cent per gallon would raise one billion dollars per year at current consumption rates. Since a tax of fifty cents per gallon would therefore raise fifty billion dollars per year, it seems a perfect way to deal with the federal budget deficit. This tax would have the additional advantage that the resulting drop in the demand for gasoline would be ecologically sound and would keep our county from being too dependent on foreign oil producers.

Which one of the following most clearly identifies an error in the author’s reasoning?

A. The author cites irrelevant data.
B. The author relies on incorrect current consumption figures.
C. The author makes incompatible assumptions.
D. The author mistakes an effect for a cause.
E. The author appeals to conscience rather than reason.

参考答案:C
思路:作者得出的结论有两个,1是提高汽油税率,可以同比提高税收所得,这一结论的前提条件是,汽油的消费量不受税率影响;2是提高税率可以降低对汽油的消费需求,这一结论的假设条件与前一结论的假设条件是互相矛盾的,因此显而易见,C正确。

Test 5
作者: capsicum    时间: 2011-7-8 10:14

73. There is no reason why the work of scientists has to be officially confirmed before being published. There is a system in place for the confirmation or disconfirmation of scientific findings, namely, the replication of results by other scientists. Poor scientific work on the part of any one scientists, which can include anything from careless reporting practices to fraud, is not harmful. It will be exposed and rendered harmless when other scientists conduct experiments and obtain disconfirmatory results.

Which one of the following, if true, would weaken the argument?

A. Scientific experiments can go unchallenged for many years before they are replicated.
B. Most scientists work in universities, where their work is submitted to peer review before publication.
C. Most scientists are under pressure to make their work accessible to the scrutiny of replication.
D. In scientific experiments, careless reporting is more common than fraud.
E. Most scientists work as part of a team rather than alone.

答案:A
思路:作者说no必要有一个新的出版confirmation系统,因为Poor scientific work is not harmful,other scientists conduct experiments来正明正确的结果。A指出Scientific experiments can go unchallenged for many years,指出Poor scientific work is harmful,是weaken。

A. 无关
B. 无关
C. 无关
D. 无关
作者: capsicum    时间: 2011-7-8 10:15

74. Governments have only one response to public criticism of socially necessary services: regulation of the activity of providing those services. But governments inevitably make the activity more expensive by regulating it, and that is particularly troublesome in these times of strained financial resources. However, since public criticism of childcare services has undermined all confidence in such services, and since such services are socially necessary, the government is certain to respond.

Which one of the following statements can be inferred from the passage?

A. The quality of child-care will improve.
B. The cost of providing child-care services will increase.
C. The government will use funding to foster advances in child-care.
D. If public criticism of policy is strongly voiced, the government is certain to respond.
E. If child-care services are not regulated, the cost of providing childcare will not increase.

A. 无关信息
B. 有效的建立了关系:government respond --- > regulation --- > make the activity
more expensive troublesome --- > troublesome in financial resources --- >cost increase. (B )
C. 无关信息:
D. 无关信息
E. 是否命题, 不能得到的结论
作者: capsicum    时间: 2011-7-8 10:15

75. Advertisers are often criticized for their unscrupulous manipulation of people’s tastes and wants. There is evidence, however, that some advertisers are motivated by moral as well as financial considerations. A particular publication decided to change its image from being a family newspaper to concentrating one sex and violence, thus appealing to a different readership. Some advertisers withdrew their advertisements from the publication, and this must have been because they morally disapproved of publishing salacious material.

Which one of the following, if true, would most strengthen the argument?

A. The advertisers switched their advertisements to other family newspapers.
B. Some advertisers switched from family newspapers to advertise in the changed publication.
C. The advertisers expected their product sales to increase if they stayed with the changed publication, but to decrease if they withdrew.
D. People who generally read family newspapers are not likely to buy newspapers that concentrate on sex and violence.
E. It was expected that the changed publication would appeal principally to those in a different income group.

答案:C
思路:
A. 支持但不是最强
B. advertisers考虑了publication转变了不同的读者群,还是financial considerations。
C. 直接举证说明原因:是为了moral,应为sales会increase if they stayed with the changed publication,但sales会decrease if they withdrew。
D. 无关
E. 无关
作者: capsicum    时间: 2011-7-8 10:16

76. Learning how to build a nest plays an important part in the breeding success of birds. For example, Dr. Snow has recorded the success of a number of blackbirds in several successive years. He finds that birds nesting for the first time are less successful in breeding than are older birds, and also less successful than they themselves are a year later. This cannot be a mere matter of size and strength, since blackbirds, like the great majority of birds, are fully grown when they leave the nest. It is difficult to avoid the conclusion that they benefit by their nesting experience.

Which one of the following, if true, would most weaken the argument?

A. Blackbirds build better nests than other birds.
B. The capacity of blackbirds to lay viable eggs increase with each successive trial during the first few years of reproduction.
C. The breeding success of birds nesting for the second time is greater than that of birds nesting for the first time.
D. Smaller and weaker blackbirds breed just as successfully as bigger and stronger blackbirds.
E. Up to 25 percent of all birds are killed by predators before they start to nest.

参考答案:B
思路:结论:How to build a nest is very important.第一句话。论据:birds nesting ,不如老bird success, also less than 。。。其实就一句话,新birds没用经验nesting,所以生育成功率低。
削弱方向:
1、断桥,论据与结论无关,即新birds nesting。。 推不出how to build a nest plays an important part in the breeding success of birds.
2、他因:有其他的原因可以解释论据。
3、驳斥论据的有效性;

A. 无关,比较点不对,和其他bird无关
B. 他因,success 是与capacity 有关。
C. 无关,陷阱,把论据重新说呢一遍。The breeding success of birds nesting for the second time is greater than that of birds nesting for the first time.= He finds that birds nesting for the first time are less successful in breeding than are older birds, and also less successful than they themselves are a year later.
D. 无关,文中没有比较smaller ,weaker。Smaller, weaker birds 完全有可能知道 how to build a nest .
E. 无关,birds被杀和breeding success of birds 无关,如果是出生(breeding)的birds 被杀就有关了。

评论:排除法是最有效的,因为不正确选项简单。而正确选项的推理过程有点扰。
作者: capsicum    时间: 2011-7-8 10:17

77. How do the airlines expect to prevent commercial plane crashes? Studies have shown that pilot error contributes to two-thirds of all such crashes. To address this problem, the airline have upgraded their training programs by increasing the hours of classroom instruction and emphasizing communication skills in the cockpit. But it is unrealistic to expect such measures to compensate for pilots’ lack of actual flying time. Therefore, the airlines should rethink their training approach to reducing commercial crashes.

Which one of the following is an assumption upon which the argument depends?

A. Training programs can eliminate pilot errors.
B. Commercial pilot routinely undergo additional training throughout their careers.
C. The number of airline crashes will decrease if pilot training programs focus on increasing actual flying time.
D. Lack of actual flying time is an important contributor to pilot error in commercial plane crashes.
E. Communication skills are not important to pilot training programs.

参考答案:D
思路:xxx的program 不能补充pilots’ lack of actual flying time.结论:thererfore, rethink approach to prevent commercial plane crash. 中间有思维跳跃,actual flying time 与plane crash 什么关系?
A. 削弱,program can eliminate…, so do not need rethink approach
B. 无关。题中是prevent planes crashes的training.
C. 加强。
D. 将actual flying time 与(pilot error in) plane crashes 联系起来。对assumption取非:actual flying time is not an important contributor to pilot error in commercial plane crashes. 削弱结论。符合ETS假设定义。
E. 加强。

评论:比较简单,不要关心废话。


Questions 78-79
Despite improvements in treatment for asthma, the death rate from this disease has doubled during the past decade from its previous rate. Two possible explanations for this increase have been offered. First, the recording of deaths due to asthma has become more widespread and accurate in the past decade than it had been previously. Second, there has been an increase in urban pollution. However, since the rate of deaths due to asthma has increased dramatically even in cities with long-standing, comprehensive medical records and with little or no urban pollution, one must instead conclude that the cause of increased deaths is the use of bronchial inhalers by asthma sufferers to relieve their symptoms.
作者: capsicum    时间: 2011-7-8 10:18

78. Each of the following, if true, provides support to the argument EXCEPT:

A. Urban populations have doubled in the past decade.
B. Records of asthma deaths are as accurate for the past twenty years as for the past ten years.
C. Evidence suggests that inhalers make the lungs more sensitive to irritation by airborne pollen.
D. By temporarily relieving the symptoms of asthma, inhalers encourage sufferers to avoid more beneficial measures.
E. Ten years ago bronchial inhalers were not available as an asthma treatment.

参考答案:A
思路:现象:death rate doubled. 结论:the cause of increased death is ……
支持方向:
1,论据的有效性。
2,的确是这个原因。
A. 削弱, 人口double了,支持了two possible explanations之一,反对了结论。
B. 支持。中心,过去,现在一样accurate.
C. 支持结论,就是inhalers 造成的。
D. 同C。
E. 支持,排除可以削弱的他因(eg, Ten years ago bronchial inhalers were as many as that used today. 就是削弱)。
作者: capsicum    时间: 2011-7-8 10:19

79. Which one of the following is an assumption on which the argument depends?

A. Urban pollution has not doubled in the past decade.
B. Doctors and patients generally ignore the role of allergies in asthma.
C. Bronchial inhalers are unsafe, even when used according to the recommended instructions.
D. The use of bronchial inhalers aggravates other diseases that frequently occur among asthma sufferers and that often lead to fatal outcomes even when the asthma itself does not.
E. Increased urban pollution, improved recording of asthma deaths, and the used of asthma deaths, and the use of bronchial inhalers are the only possible explanations of the increased death rate due to asthma.

参考答案:E
思路:不是解释1和2,就是解释3。隐含了1,2,3外没用其他解释的理由。
A. 支持。
B. 无关。中心是role的问题。
C. 可以算无关,也可以算削弱。
D. 加强
E. 取非,除了1,2,3解释还有其他解释。典型的他因削弱,符合ETS 假设定义。
作者: capsicum    时间: 2011-7-8 10:19

80. Advertisement: Clark brand-name parts are made for cars manufactured in this country. They satisfy all of our government automotive tests-the toughest such tests in the world. With foreign-made parts, you never know which might be reliable and which are cheap look-alikes that are poorly constructed and liable to cost you hundreds of dollars in repairs. Therefore, be smart and insist on brand-name parts by Clark for your car.

The argument requires the assumption that

A. Clark parts are available only in this country
B. foreign-made arts are not suitable for cars manufactured in this country
C. no foreign-made parts satisfy our government standards
D. parts that satisfy or government standards are not as poorly constructed as cheap foreign made parts
E. if parts are made for cars manufactured in our country, they are not poorly constructed

参考答案:D
思路:C东东通过了G test, 所以,选C东东,不选国外东东。
A. 无关,而且有only,
B. 加强。
C. 无关。用不用G和国外的与satisfy standards 无关。
D. 取非。通过了test的G与国外的一样。削弱。
E. 加强。
作者: capsicum    时间: 2011-7-8 10:19

81. Nuclear fusion is a process whereby the nuclei of atoms are joined, or “fused”, and in which energy is released. One of the by-products of fusion is helium-4 gas. A recent fusion experiment was conducted using “heavy” water contained in a sealed flask. The flask was, in turn, contained in an air-filled chamber designed to eliminate extraneous vibration. After the experiment, a measurable amount of helium-4 gas was found in the air of the chamber. The experiments cited this evidence in support of their conclusion that fusion had been achieved.

Which one of the following, if true, would cast doubt on the experimenters’ conclusion?

A. Helium-4 was not the only gas founding the experiment chamber.
B. When fusion is achieved, it normally produces several by-products, including tritium and gamma rays.
C. The amount of helium-4 found in the chamber’s air did not exceed the amount of helium-4 that is found in ordinary air.
D. Helium-4 gas rapidly breaks down, forming ordinary helium gas after a few hours.
E. Nuclear fusion reactions are characterized by the release of large amount of heat.

参考答案:C
思路:FACT: nuclear fusion产生H-4. 现象:某个试验有he-4,结论:试验中有nuclear fusion.

驳斥方向:
1. 断桥。有h4推不出nuclear fusion,不过本题无法断桥,因为这是个Fact.
2. 他因,试验中的h4有其他来源。

A. 无关
B. 无关,和A说的同一件事情。
C. 他因。试验中h4 是本来就有的。
D. 无关,本句中心是helium gas 不是h-4。Helium 属于文中没用提的东东,虽然都是helium.
E. 无关,陷阱。试验中完全可能有大量的热,作者没用说,谁也不知道有没有热。
作者: capsicum    时间: 2011-7-8 10:20

82. Every photograph, because it involves the light rays that something emits hitting film, must in some obvious sense be true. But because it could always have been made to show things differently than it does, it cannot express the whole truth and, in that sense, is false. Therefore, nothing can ever be definitively proved with a photograph.

Which one of the following is an assumption that would permit the conclusion above to be properly drawn?

A. Whatever is false in the sense that it cannot express the whole truth cannot furnish definitive proof.
B. The whole truth cannot be known.
C. It is not possible to determine the truthfulness of a photograph in any sense.
D. It is possible to use a photograph as corroborative evidence if there is additional evidence establishing the truth about the scene photographed.
E. If something is being photographed, then it is possible to prove definitively the truth about it.

参考答案:A
思路:
前提:the photograph in some obvious sense be true, but it cannot express the whole truth .
推论:nothing can ever be definitively proved with a photograph. The problem is which of the following can permit the conclusion be properly drawn.

A. what cannot express the whole truth cannot furnish definitive proof. Right!
B. it is obviously wrong
C. sometimes the photograph cannot express the whole truth but not in any sense. Wrong.
D. this sentence said that photograph can use as an extra evidence for the truth, but it is no relations with the problem.
E. it is obviously wrong according to the passage.
作者: capsicum    时间: 2011-7-8 10:20

Questions 83-84
Some cleaning fluids, synthetic carpets, wall paneling, and other products release toxins, such as formaldehyde and benzene, into the household air supply. This is not a problem in well-ventilated houses, but it is a problem in houses that are so well insulated that they trap toxins as well as heat. Recent tests, however, demonstrate that houseplants remove some household toxins from the air and thereby eliminate their danger. In one test, 20 large plants eliminated formaldehyde from a small, well-insulated house.

83. Assume that a person who lives in a small, well-insulated house that contains toxin-releasing products places houseplants, such as those tested, in the house.

Which one of the following can be expected as a result?

A. There will no longer be any need to ventilate the house.
B. The concentration of toxins in the household air supply will remain the same.
C. The house will be warm and have a sage air supply.
D. If there is formaldehyde in the household air supply, its level will decrease.
E. If formaldehyde benzene are being released of each will decrease.

参考答案:D
思路:the passage point out that something will release toxins into the air. This is not a problem in well-ventilated houses but a problem in insulated houses. But houseplants can remove some household toxins from the air. In one test 20 large plants eliminated one kind of toxins from a small well-insulated house.

The problems is a person live in a small ,well-insulated house that contains toxin-releasing products. We places some houseplants in the house as those tested. Which one can be a result?

A. we can not include from the passage that houseplants can eliminate all kinds of toxins from the house so ventilation is needed in some way.
B. formaldehyde will be eliminated and the concentration of toxins won’t remain the same.
C. make reference to A
D. it can conclude from the passage! Right!
E. make reference to A
作者: capsicum    时间: 2011-7-8 10:21

84. The passage is structured to lead to which one of the following conclusions?

A. Houseplants can remove benzene from the air.
B. Nonsynthetic products do not release toxins into houses.
C. Keeping houseplants is an effective means of trapping heat in a poorly insulated house.
D. Keeping houseplants can compensate for some of the negative effects of poor ventilation.
E. The air in a well-insulated house with houseplants will contain fewer toxins than the air in a well-ventilated house without houseplants.

参考答案:D
思路:the problem is which one of the following is truth concluded from the passage!
A. we can not include from the passage that houseplants can eliminate all kinds of toxins from the house. Benzene is not mentioned above. Wrong!
B. the passage does not mention which kind product don’t release toxins. Wrong!
C. including from the passage that houseplants can eliminate one kind of toxins , formaldehyde ,from the house but can not trap heat. Wrong!
D. right! Ventilation is better than keeping houseplants in a house.
E. wrong, just the opposite.


Questions 85-86
In opposing the 1970 Clean Air Act, the United States automobile industry argued that meeting the act’s standards for automobile emissions was neither economically feasible nor environmentally necessary. However, the catalytic converter, invented in 1967, enabled automakers to meet the 1970 standards efficiently. Currently, automakers are lobbying against the government’s attempt to pass legislation that would tighten restrictions on automobile emissions. The automakers contend that these new restrictions would be overly expensive and unnecessary to efforts to curb air pollution. Clearly, the automobile industry’s position should not be heeded.
作者: capsicum    时间: 2011-7-8 10:21

85. Which one of the following most accurately expresses the method used to counter the automakers’ current position?

A. The automakers’ premises are shown to lead to a contradiction.
B. Facts are mentioned that show that the automakers are relying on false information.
C. A flaw is pointed out in the reasoning used by the automakers to reach their conclusion.
D. A comparison is drawn between the automakers’ current position and a position they held in the past.
E. Evidence is provided that the new emissions legislation is both economically feasible and environmentally necessary.

The automakers use the reason that these new restrictions would be overly expensive. However, in the past, the automakers previously also used the same reason and their position is proved to be wrong. Consequently the passage draws a comparison, that is, an analogy to disprove the reasoning of the automakers. Thus, D, a comparison is drawn between the automakers’ current position and a position they held in the past, is the most accurate and thus the best choice.

A. The automakers’ premises are not shown to lead a contradiction. Actually if the premises are true, they support their standing point.
B. There is no fact to show that the automakers are relying on false information.
C. The automakers’ reasoning itself does not contain error. The reasoning could only be shown not correct when confronted with past case.
E. There is no evidence that is provided for such purpose here in the passage.

86. Which one of the following, if true, lends the most support to the automakers’ current position?

A. The more stringent the legislation restricting emission becomes, the more difficult it becomes for automakers to provide the required technology economically.
B. Emissions-restriction technology can often be engineered so as to avoid reducing the efficiency with which an automobile uses fuel.
C. Not every new piece of legislation restricting emission requires new automotive technology in order for automakers to comply with it.
D. The more automobiles there are on the road, the more stringent emission restrictions must be to prevent increased overall air pollution.
E. Unless forced to do so by the government, automakers rarely make changes in automotive technology that is not related to profitability.

The automakers’ currently contends that these new restrictions would be overly expensive. If the situation is still the same, if there is no increased difficulty for the automakers to reach the difficulty, the reason here mentioned by them would be simply a scapegoat here for them to escape the necessary effort. However, if the truth is as said in choice A, the more stringent the legislation restricting emission becomes, the more difficult it becomes for automakers to provide the required technology economically, which means that because the current situation asks for even more advanced technology and the increased difficulty could not be overcome economically, the protest of the automakers is well supported.

B. actually weakens the automakers’ current position
C. it has nothing to do with supporting the automakers’ current position. Actually this claim could only express that the automakers do not need to protest that much.
D. If provided together with other information, such as “there are more and more cars on the roads”, the claim of D could serve as a weakening the automakers’ current position.
E. It only makes clear about why necessary for government to intervene and how to realize the plan about reducing the air pollution; in fact, it does not weaken or strengthen the automakers’ current position.
作者: capsicum    时间: 2011-7-8 10:21

87. A survey was recently conducted among ferry passengers on the North Sea. Among the results was this: more of those who had taken anti-seasickness medication before their trip reported symptoms of seasickness than those who had not taken such medication. It is clear, then, that despite claims by drug companies that clinical tests show the contrary, people would be better off not taking anti-seasickness medications.

Which one of the following, if true, would most weaken the conclusion above?

A. Given rough enough weather, most ferry passengers will have some symptoms of seasickness.
B. The clinical tests reported by the drug companies were conducted by the drug companies’ staffs.
C. People who do not take anti-seasickness medication are just as likely to respond to a survey on seasickness as people who do.
D. The seasickness symptoms of the people who took anti-seasickness medication would have been more severe had they not taken the medication.
E. People who have spent money on anti-seasickness medication are less likely to admit symptoms of seasickness than those who have not.

The passage concludes that the medicine is not useful at all because the people taking the drug appear to be sicker than those people not taking the drug. However, it does not consider the people taking the drug could appear to be even sicker if they do not take the drug. It does not make the wise comparison between two originally the same survey group. So D most accurately point out the lope hole in the reasoning of the passage, thus the best answer.

A. By eliminating the difference of the two survey groups, actually support the conclusion.
B. Without further information, the conclusion will not be changed a little whoever conducted the test.
C. By eliminating the difference of the two survey groups, weakly support the conclusion.
D. As discussed.
E. By even further decrease the sampling difference, E actually support the conclusion.
作者: capsicum    时间: 2011-7-8 10:22

88. Court records from medieval France show that in the years 1300 to 1400 the number of people arrested in the French realm for “violent interpersonal crimes” (not committed in wars) increased by 30 percent over the number of people arrested for such crimes in the years 1200 to 1300. If the increase was not the result of false arrests, therefore, medieval France had a higher level of documented interpersonal violence in the years 1300 to 1400 than in the years 1200 to 1300.

Which one of the following statements, if true, most seriously weakens the argument?

A. In the years 1300 to 1400 the French government’s category of violent crimes included an increasing variety of interpersonal crimes that are actually nonviolent.
B. Historical accounts by monastic chroniclers in the years 1300 to 1400 are filled with descriptions of violent attacks committed by people living in the French realm.
C. The number of individual agreements between two people in which they swore oaths not to attack each other increased substantially after 1300.
D. When English armies tried to conquer parts of France in the mid-to late 1300s, violence in the northern province of Normandy and the southwestern province of Gascony increased.
E. The population of medieval France increased substantially during the first five decades of the 1300s, until the deadly bubonic plague decimated the population of France after 1348.
|
参考答案:A
思路:他因削弱。
A. “1300-1400年间,法国政府的暴力罪范畴将越来越多的非暴力人际关系罪包括在内。”
B. “在有关1300-1400的历史记述中,充满了关于法国境内居民进行暴力攻击的描述。”无关选项。
C. “1300年以后,二人之间立誓不去互相攻击对方的个人协议大量增加。”无关选项,不能说明犯罪案件是否增加(协议增加可能会使犯罪减少,也可能因为犯罪增加导致协议增多)
D. “1300年中期至末期,在英国军队试图征服法国时,诺曼底北部省份及加斯科涅西南部省份的暴力犯罪有所上升” 支持结论
E. “致命的黑死病使法国人口在1348后大量减少,在此之前,14世纪前50年中法国人口大量增加” 无关选项,人口增加与人际关系犯罪案件上升无必然联系。
作者: capsicum    时间: 2011-7-8 10:23

89. Rhizobium bacteria living in the roots of bean plants or other legumes produce fixed nitrogen, which is one of the essential plant nutrients and which for non-legume crops, such as wheat, normally must be supplied by applications of nitrogen-based fertilizer. So if biotechnology succeeds in producing wheat strains whose roots will play host to Rhizobium bacteria, the need for artificial fertilizers will be reduced.

The argument above makes which one of the following assumptions?

A. Biotechnology should be directed toward producing plants that do not require artificial fertilize.
B. Fixed nitrogen is currently the only soil nutrient that must be supplied by artificial fertilizer for growing wheat crops.
C. There are no naturally occurring strains of wheat or other grasses that have Rhizobium bacteria living in their roots.
D. Legumes are currently the only corps that produce their own supply of fixed nitrogen.
E. Rhizobium bacteria living in the roots of wheat would produce fixed nitrogen.

参考答案:E
思路:原文的结论是“如果生物工程可以成功培育出能寄生根瘤菌的小麦品种,对人工肥料的需求就会减少”;“生活在大豆或其它豆科植物根部的根瘤菌产生固氮(一种重要的植物养份)”是结论的major premise,应该在二者之间搭桥建立联系,即小麦上寄生的根瘤菌同样能产生固氮。
作者: capsicum    时间: 2011-7-8 10:23

90. Current legislation that requires designated sections for smokers and nonsmokers on the premises of privately owned businesses is an intrusion into the private sector that cannot be justified. The fact that studies indicate that nonsmokers might be harmed by inhaling the smoke from others’ cigarettes is not the main issue. Rather, the main issue concerns the government’s violation of the right of private businesses to determine their own policies and rules.

Which of the following is a principle that, if accepted, could enable the conclusion to be properly drawn?

Government intrusion into the policies and rules of private businesses is justified only when individuals might be harmed.
The right of individuals to breathe safe air supersedes the right of businesses to be free from government intrusion.
The right of businesses to self-determination overrides whatever right or duty the government may have to protect the individual.
It is the duty of private businesses to protect employees from harm in the workplace.
Where the rights of businesses and the duty of government conflict, the main issue is finding a successful compromise.

Current legislation that requires designated sections for smokers and nonsmokers on the premises of privately owned businesses is an intrusion into the private sector that cannot be justified (conclusion from the argument). The fact that studies indicate that nonsmokers might be harmed by inhaling the smoke from others’ cigarettes is not the main issue (refute a possible reason that could be used to counter the conclusion). Rather, the main issue concerns the government’s violation of the right of private businesses to determine their own policies and rules (induce reason for the conclusion to be drawn).

A,反对原文论证
B,反对原文论证
C,答案,it makes clear that the right of business is more important than the duty for government to protect individual, building a bridge for the conclusion to be drawn.
D,无关
E,无关

TEST 6
作者: capsicum    时间: 2011-7-8 10:24

91. Leachate is a solution, frequently highly contaminated, that develops when water permeates a landifill’s site. If and only if the landfill’s capacity to hold liquids is exceeded does the leachate escape into the environment, generally in unpredictable quantities. A method must be found for disposing of leach ate. Most landfill leachate is sent directly to sewage treatment plants, but not all sewage plants are capable of handling the highly contaminated water.

Which one of the following can be inferred from the passage?

A. The ability to predict the volume of escaping landfill leachate would help solve the disposal problem.
B. If any water permeates a landfill, leachate will escape into the environment.
C. No sewage treatment plants are capable of handling leachate.
D. Some landfill leachate is sent to sewage treatment plants that are incapable of handling it.
E. If leachate does not escape from a landfill into the environment, then the landfill’s capacity to hold liquids has not been exceeded.

此题为充要条件的逆否命题。
A是B的充要条件,那末他的逆否命题比正确! 非B推出非A

If and only if the landfill’s capacity to hold liquids is exceeded does the leachate escape into the environment, ---------〉If leachate does not escape from a landfill into the environment, then the landfill’s capacity to hold liquids has not been exceeded.

参考答案E
思路:原文说:“Leachate(浸析液)是一种水渗透过垃圾填埋地后生成的溶液,这种溶液通常是高度污染的。当且仅当超出垃圾填埋的容水能力时(A),Leachate会泄露到外界环境中(B)。必须找到一种处理L的方法。许多L被送去用于污水灌溉,但不是所有的污水灌溉的植物都有能力处理这种被高度污染的水”。
原文中If and only if A then B(从B可推出A);选项E为If not A then not B(非A推非B)正确

A. 原文没有说
B. 条件不对
C. 原文是一部分plants没有这种能力
D. 原文只说明并非所有的污水灌溉的植物都有能力处理这种被高度污染的水,没说some Leachate一定是被送去浇那些无此能力的植物。
作者: capsicum    时间: 2011-7-8 10:24

92. Baking for winter holidays is a tradition that may have a sound medical basis. In midwinter, when days are short, many people suffer from a specific type of seasonal depression caused by lack of sunlight. Carbohydrates, both sugars and starches, boost the brain’s levels of serotonin, a neurotransmitter that improves the mood. In this respect, carbohydrates act on the brain in the same way as some antidepressants. Thus, eating holiday cookies may provide an effective form of self-prescribed medication.

Which one of the following can be properly inferred from the passage?

A. Seasonal depression is one of the most easily treated forms of depression.
B. Lack of sunlight lowers the level of serotonin in the brain.
C. People are more likely to be depressed in midwinter than at other times of the year.
D. Some antidepressants act by changing the brain’s level of serotonin.
E. Raising the level of neurotransmitters in the brain effectively relieves depression.

从这句话, boost the brain’levels of serotonin, a neurotransmitter that improves the mood, 得知, 只是serotonin有抗忧郁的作用, 而不是所有的neurotransmitter.
D is correct.
A,most easily treated forms和原文无关
B,没有提到的无关信息
C,无关比较
E,把原文的信息扩大范围了,原文说的是a neurotransmitter, E中变成了所有neurotransmitters。


93. The efficiency of microwave ovens in destroying the harmful bacteria frequently found in common foods is diminished by the presence of salt in the food being cooked. When heated in a microwave oven, the interior of unsalted food reaches temperatures high enough to kill bacteria that cause food poisoning, but the interior of salted food does not. Scientists theorize that salt effectively blocks the microwaves from heating the interior.

Which one of the following conclusions is most supported by the information above?

A. The kinds of bacteria that cause food poisoning are more likely to be found on the exterior of food than in the interior of food.
B. The incidence of serious food poisoning would be significantly reduced if microwave ovens were not used by consumers to cook or reheat food.
C. The addition of salt to food that has been cooked or reheated in a microwave oven can increase the danger of food poisoning.
D. The danger of food poisoning can be lessened if salt is not used to prepare foods that are to be cooked in a microwave oven.
E. Salt is the primary cause of food poisoning resulting from food that is heated in microwave ovens.

In my view, the reasoning in this argument is that:
On the one hand:
the presence of salt in the food being cookedà salt effectively blocks the microwaves from heating the interiorà The efficiency of microwave ovens in destroying the harmful bacteria frequently found in common foods is diminished
On the other hand:
unsalted food is heated by microwaveà the interior of unsalted food reaches temperatures high enoughà kill bacteria that cause food poisoning

So , D is the best choice.

A. is out of scope because the exterior of food is not mentioned in this argument.
B. is also out of scope too. We never know what will happen if microwave ovens were not used by consumers to cook or reheat food. Because the statement have no the relevant information.
C. is puzzle. The statement just infers that the harmful bacteria frequently found in common foods is keep same. The salt only prevent microwave from destroying such bacteria. Even though the addition of salt to food will significantly prevent the efficiency of microwave ovens, we can not reach the conclusion that the addition of salt to food that has been cooked or reheated in a microwave oven can increase the danger of food poisoning. So C is problematic choice.
D. is the best choice.
E. may be the extreme pole. The statement not mention that salt is the primary cause of food poisoning resulting from food. It is possible that salt is the relevant factor in determining the efficiency of microwave ovens.
作者: capsicum    时间: 2011-7-8 10:25

94. When 100 people who have not used cocaine are tested for cocaine use, on average only 5 will test positive. By contrast, of every 100 people who have used cocaine 99 will test positive. Thus, when a randomly chosen group of people is tested for cocaine use, the vast people is tested for cocaine use, the vast majority of those who test positive will be people who have used cocaine.

A reasoning error in the argument is that the argument

A. attempts to infer a value judgment from purely factual premises
B. attributes to every member of the population the properties of the average member of the population
C. fails to take into account what proportion of the population have used cocaine
D. ignores the fact that some cocaine users do not test positive
E. advocates testing people for cocaine use when there is no reason to suspect that they have used cocaine

CR will get more interesting when math is involved. The most seen math problem in GMAT CR is percentage. What ETS wants to test is whether you can distinguish between absolute value and relative value, e.g. percentage.
What you need to remember is that higher percentage in CR does not indicate that the absolution number or quantity will be higher compared to lower percentage. To the contrary, higher percentage always has smaller numbers or quantity.
In this question, the conclusion is the last sentence. The premises are that 5% of non-users and 99% of users are tested positive. The argument fails to consider the composition of population and makes an unwarranted assumption that among a random group of people, non-users are less than 99/5 of users. X: non-users Y: users. 5%X < 99%Y è x<99/5Y. If this proportion is not true, the argument does not hold.
So you need to know whether the question is about absolute value or relative value. If it is about relative value, you need to know what the BASE is. If comparison is involved, you need to find out the relationship between the bases.
作者: capsicum    时间: 2011-7-8 10:25

95. Scientific research that involves international collaboration has produced papers of greater influence, as measured by the number of times a paper is cited in subsequent papers, than has research without any collaboration. Papers that result from international collaboration are cited an average of seven times, whereas papers with single authors are cited only three times on average. This difference shows that research projects conducted by international research teams are of greater importance than those conducted by single researchers.

Which one of the following is an assumption on which the argument depends?

A. Prolific writers can inflate the number of citations they receive by citing themselves in subsequent papers.
B. It is possible to ascertain whether or not a paper is the product of international collaboration by determining the number of citations it has received.
C. The number of citations a paper receives is a measure of the importance of the research it reports.
D. The collaborative efforts of scientists who are citizens of the same country do not produce papers that are as important as papers that are produced by international collaboration.
E. International research teams tend to be more generously funded than are single researchers.

In my view, the reasoning in this argument is that:
Papers that result from international collaboration are cited an average of seven times, whereas papers with single authors are cited only three times on average.à research projects conducted by international research teams are of greater importance than those conducted by single researchers.

The hidden assumption is that we can evaluate the importance of research projects by the number of times a paper is cited in subsequent papers.

Obviously, C is the best answer.

A. is out of scope. We can also have good reasons to doubt whether the prolific writers is representative enough to reflect the whole situation of the single authors.
B. the same mistake. Even we can ascertain whether or not a paper is the product of international collaboration by determining the number of citations it has received. The preceding assumption have no effect on the conclusion.
C. is the best answer.
D. is absurd. We have not known anything about the domestic or the alien authors.
E. is out of scope too. There is nothing relevant to fund in this argument.
作者: capsicum    时间: 2011-7-8 10:25

96. Prominent business executives often play active roles in United States presidential campaigns as fundraisers or backroom strategists, but few actually seek to become president themselves. Throughout history the great majority of those who have sought to become president have been lawyers, military leaders, or full-time politicians. This is understandable, for the personality and skills that make for success in business do not make for success in politics. Business is largely hierarchical, whereas politics is coordinative. As a result, business executives tend to be uncomfortable with compromises and power-sharing, which are inherent in politics.

Which one of the following, if true, most seriously weakens the proposed explanation of why business executives do not run for president?

A. Many of the most active presidential fundraisers and backroom strategists are themselves politicians.
B. Military leaders are generally no more comfortable with compromises and power-sharing than are business executives.
C. Some of the skills needed to become a successful lawyer are different from some of those needed to become a successful military leader.
D. Some former presidents have engaged in business ventures after leaving office.
E. Some hierarchically structured companies have been major financial supporters of candidates for president.

In my view, the reasoning in this argument is that:
business executives tend to be uncomfortable with compromises and power-sharing, which are inherent in politics  few actually seek to become president themselves.

The hidden assumption is that:
lawyers, military leaders, or full-time politicians tend to be comfortable with compromises and power-sharing.

How to weaken this explanation?
In my view, we can undermine this reasoning in this argument in two ways. On the one hand, we can find other reasons excepting for the listed explanation, such as business executives worry about the safe of the presidents, or business executives enjoy freedom, etc. On the other hand, we can rule out analogue between business executives and the listed candidate including lawyers, military leaders, or full-time politicians. That is to say, if we can sure that some of such candidates are also tired of compromises and power sharing like business executives. The reasoning will be seriously undermined.

After this analysis, we can choose B quickly.


97. A scientific theory is a good theory if it satisfies two requirements: it must accurately describe a large class of observations in terms of a model that is simple enough to contain only a few elements, and it must make definite predictions about the results of future observations. For example, Aristotle’s cosmological theory, which claimed that everything was made out of four elements---earth, air, fire, and water---satisfied the first requirement, but it did not make any definite predictions. Thus, Aristotle’s cosmological theory was not a good theory.

If all the statements in the passage are true, each of the following must also be true EXCEPT:

A. Prediction about the results of future observations must be made by any good scientific theory
B. Observation of physical phenomena was not a major concern in Aristotle’s cosmological theory.
C. For elements can be the basis of a scientific model that is simple enough to meet the simplicity criterion of a good theory.
D. A scientific model that contains many elements is not a good theory.
E. Aristotle’s cosmological theory described a large class of observations in terms of only four elements.

After analysis , we can find that the reasoning in this argument is that:
1> it must accurately describe a large class of observations in terms of a model that is simple enough to contain only a few elements and 2> it must make definite predictions about the results of future observations.&szlig;à A scientific theory is a good theory

So , we can find all of choices except B satisfied with the original meaning.

B is not mentioned in this argument , so we can conclude whether B is also true.
So B is the best answer.
作者: capsicum    时间: 2011-7-8 10:26

98. Compared to nonprofit hospitals of the same size, investor-owned hospitals require less public investment in the form of tax breaks, use fewer employees, and have higher occupancy levels. It can therefore be concluded that investor-owned hospitals are a better way of delivering medical care than are nonprofit hospitals.

Which one of the following, if true, most undermines the conclusion drawn above?

A. Nonprofit hospitals charge more per bed than do investor-owned hospitals.
B. Patients in nonprofit hospitals recover more quickly than do patients with comparable illnesses in investor-owned hospitals.
C. Nonprofit hospitals do more fundraising than do investor-owned hospitals.
D. Doctors at nonprofit hospitals earn higher salaries than do similarly-qualified doctors at investor-owned hospitals.
E. Nonprofit hospitals receive more donations than do investor-owned hospital.

After analysis of the above statement, we can make it clear that the arguer want to establish the casual relationship between the listed factors, including less public investment in the form of tax breaks, fewer employees and higher occupancy levels, and a better way of delivering medical care.
How to undermine this reasoning?
In my view, I think that we can weaken this argument in two ways. On the one hand, we can destroy the casual relationship the arguer wants to create. On the one hand, we can cite the fact that opposite the conclusion the arguer want to come to.

So , taking into account the above analysis , B is sound.
Because B cites the fact opposing the conclusion.

A is out of scope. The statement does not mention the relationship between the number of per bed and the better way of delivering medical care.

C makes the same mistake. The relationship between the fundraising and the better way of delivering medical care.

D is repeating the above flaw. The reasoning that the higher salaries doctors received, the better way of delivering medical care is open to doubt.

Finally, E follows the same way again. The arguer fails to convince us that more donations will lead to the better way of delivering medical care.
作者: capsicum    时间: 2011-7-8 10:34

68. Since no one returns from death, we can never be certain about what passes through the mind of the dying person. For the unconscious, the confused, and the heavily sedated, these final moments are probably meaningless. However, for the mentally alert, it is quite possible that death presents itself as an unbelievably glorious experience, a flight into an entirely new universe of sensation. Why should we think so? Some people who have been reprieved from “certain” death at the last moment have experienced what goes through the consciousness of those who are not so fortunate. For example, parachutists who have survived falls report experiences that resemble psychedelic “trips”.

The primary point of the argument in the passage is

A. no one returns from death
B. dying can be a glorious experience
C. we can never know what passes through the mind of a dying person
D. some people are reprieved from death at the last moment
E. some people “die”, yet live to report their experiences

答案:B
思路:问题问短文论点。短文在第三行出现However,其前为一种现象的说法(不知道人死后思想上经历什么),其后强调另外一种死亡经历(Unbelievably glorious experience),后面说明后者的原因。故However引导的那句话是中心句。

A. 文中第一种说法,不正确。
B. 同However一句意思相同,正确。
C. 也是第一种说法,不正确。
D. 说明原因时使用的论据,非中心点。
E. 同D。
作者: capsicum    时间: 2011-7-8 10:34

101. Anson bases his conclusion about Dr. Ladlow on which one of the following?

A. an attack on Dr. Ladlow’s character
B. the application of a general principle
C. the use of an ambiguous term
D. the discrediting of facts
E. the rejectiion of a theoretical explanation


Questions 102-103
The brains of identical twins are genetically identical. When only one of a pair of identical twins is a schizophrenic, certain areas of the affected twin’s brain are smaller than corresponding areas in the brain of the unaffected twin. No such differences are found when neither twin is schizophrenic. Therefore, this discovery provides definitive evidence that schizophrenia is caused by damage to the physical structure of the brain.

102. Which one of the following is an assumption required by the argument?

(A) The brain of a person suffering from schizophrenia is smaller than the brain of anyone not suffering from schizophrenia.
(B) The relative smallness of certain parts of the brains of schizophrenics is not the result of schizophrenia or of medications used in its treatment.
(C) The brain of a person with an identical twin is no smaller, on average, than the brain of a person who is not a twin.
(D) When a pair of identical twins both suffer from schizophrenia, their brains are the same size.
(E) People who have an identical twin are no more likely to suffer from schizophrenia than those who do not.

答案:B
思路:
题干:assumption
原文:If one schizophrenic,then one smaller。If neither schizophrenic, then no difference. Conclusion: damage to physical structure cause schizophrenic
核心词:schizophrenic 注意:比较是在twin中进行的
我的习惯是从下到上地看选项。
E 无关项
D 原文没有的信息
C扩大了比较范围
A 扩大了比较范围
B ETS最爱:无他因
作者: capsicum    时间: 2011-7-8 10:36

103. If the statements on which the conclusion above is based are all true, each of the following could be true EXCEPT:

A. People who lack a genetic susceptibility for the disease will not develop schizophrenia
B. Medications can control most of the symptoms of schizophrenics share many of the characteristics found in those of people without the disorder
C. The brains of schizophrenics share many of the characteristics found in those of people without the disorder
D. It will eventually be possible to determine whether or not someone will develop schizophrenia on the basis of genetic information alone
E. Brain abnormalities associated with schizophrenia are the result of childhood viral infections that inhibit the development of brain cells

答案:C
题干: could be true, EXCEPT = must be false (按照老管的方法,找最确定的)
选项(从下到上):
E 完全超出原文,不能判断
C完全超出原文,不能判断
B完全超出原文,不能判断
A超出原文,不能判断
D 注意文章第一句:“genetically identical”,选项最后一词“alone”。
作者: capsicum    时间: 2011-7-8 10:37

104. That the policy of nuclear deterrence has worked thus far is unquestionable. Since the end of the Second World War, the very fact that there were nuclear armaments in existence has kept major powers from using nuclear weapons, for fear of starting a worldwide nuclear exchange that would make the land of the power initiating it uninhabitable. The proof is that a third world war between superpowers has not happened.

Which one of the following, if true, indicates a flaw in the argument?

A. Maintaining a high level of nuclear armaments represents a significant drain on a country’s economy.
B. From what has happened in the past, it is impossible to infer with certainty what will happen in the future, so an accident could still trigger a third world war between superpowers.
C. Continuing to produce nuclear weapons beyond the minimum needed for deterrence increase the likelihood of a nuclear accident.
D. The major powers have engaged in many smaller-scale military operations since the end of the Second World War, while refraining from a nuclear confrontation.
E. It cannot be known whether it was nuclear deterrence that worked, or some other factor, such as a recognition of the economic value of remaining at peace.

答案:E
题干:flaw
原文:
Conclusion:policy worked.
Evidence: end of 2 world war ; 3 world war not happen.
casual explanation.
选项:从下到上
E 他因
D 事实陈述 可以作为weaken
C 无关项
B 迷惑项 请注意:原文是从已知的事实来推测导致事实的原因,所以该事实是不能被反对的。
A 无关项
作者: capsicum    时间: 2011-7-8 10:37

105. A survey of alumni of the class of 1960 at Aurora University yielded puzzling results. When asked to indicate their academic rank, half of the respondents reported that they were in the top quarter of the graduating class in 1960.

Which one of the following most helps account for apparent contradiction above?

A. A disproportionately large number of high ranking alumni responded to the survey.
B. Few, if any, respondents were mistaken about their class rank.
C. Not all the alumni who were actually in the top quarter responded to the survey.
D. Almost all of the alumni who graduated in 1960 responded to the survey.
E. Academic rank at Aurora University was based on a number of considerations in addition to average grades.

【正确答案】A
【解题思路】
本题的类型是解释类,即解释一个客观事实或现象发生的原因。
本题所陈述的客观事实是:参加调查的一半的对象(1960年的男校友)显示他们的成绩排名处于班上的前四分之一。
本题的考点是整体与部分之间的比例关系的逻辑思维。首先可以排除的是选项C和D,选项C说不是所有排名处于前四分之一的男校友参加了调查,选项D说几乎所有的1960年的男校友参加了调查,都更不利于解释已发生的事实,即使这个客观事实发生的可能性减少。选项E说学校的排名除了考虑平均成绩外,还考虑其他很多因素,但至少还是存在排名的,因此与发生的事实风牛马不相及。只有选项A说大量的不成比例的排名在前的男校友参加了调查最好地解释的所发生的客观事实, 选项B即使可能解释所发生的客观事实,也不如A选项为优。
作者: capsicum    时间: 2011-7-8 10:38

106. The seventeenth-century physicist Sir Isaac Newton is remembered chiefly for his treatises on motion and gravity. But Newton also conducted experiments secretly for many years abased on the arcane theories of alchemy, trying unsuccessfully to transmute common metals into gold and produce rejuvenating elixirs. If the alchemists of the seventeenth century had published the results of their experiments, chemistry in the eighteenth century would have been more advance than it actually was.

Which one of the following assumptions would allow the conclusion concerning eighteenth-century chemistry to be properly drawn?

A. Scientific progress is retarded by the reluctance of historians to acknowledge the failures of some of the great scientists.
B. Advances in science are hastened when reports of experiments, whether successful or not, are available for review by other scientists.
C. Newton’s work on motion and gravity would not have gained wide acceptance if the results of his work in alchemy had also been made public.
D. Increasing specialization within the sciences makes it difficult for scientists in one field to understand the principles of other fields.
E. The seventeenth-century alchemists could have achieved their goal only if their experiments had been subjected to public scrutiny.

【正确答案】B
【解题思路】
本题的类型是假设类,即所提出的假设是结论的一个必要条件。
本题的前提是:如果17世纪的炼金术士们公开他们的实验结果
本题的结论是:那么18世纪的化学将比其原来更为先进
前提中的核心词是:实验结果。结论中的核心词是:先进。要使某一假设成为该结论的必要条件,必然要提到实验结果与先进的关系,因此本题可用找核心词的办法进行快速排除,只有选项B包含这两个核心词。




欢迎光临 国际顶尖MBA申请交流平台--TOPWAY MBA (http://forum.topway.org/) Powered by Discuz! 7.2